SBI PO 2013 Mains Previous Year Paper

SBI PO 2013 Mains

Section

Logical Reasoning

Questions

50 Questions (1 – 50)

Marks

50

Data Interpretation

50 Questions (51 – 100)

50

General Knowledge

50 Questions (101 – 150)

50

English

50 Questions (151 – 200)

50

Q. 1 Which of the following expressions will be true if the expression R > O = A > S < T is definitely true?

A. O > T

B. S < R

C. T > A

D. S = O

E. T < R

 

Q. 2 Which of the following symbols should replace the question mark (?) in the given

expression in order to make the expressions P > A as well as T≤L definitely true? P > L ? A ≥ N = T

A. ≤

B. >

C. <

D. ≥

E. Either ≤ or <

 

Q. 3 Which of the following symbols should be placed in the blank spaces respectively (in the same order from left to right) in order to complete the given expression in such a manner that makes the expressions B > N as well as D ≤ L definitely true? B _ L _ O _ N _ D

A. =, =, ≥, ≥

B. >, ≥, =, >

C. >, <, =, ≤

D. >, =, =, ≥

E. >, =, ≥, >

 

Q. 4 Which of the following should be placed in the blank spaces respectively (in the same order from left to right) in order to complete the given expression in such a manner that makes the expression A < P definitely false? _ < _ < _ > _

A. L, N, P, A

B. L, A, P, N

C. A, L, P, N

D. N, A, P, L

E. P, N, A, L

 

Q. 5 Which of the following symbols should be placed in the blank spaces respectively (in the same order from left to right) in order to complete the given expression in such a manner that makes the expression F > N and U > D definitely false? F _ O _ U _ N _ D

A. <, <, >, =

B. <, =, =, >

C. <, =, =, <

D. ≥, =, =, ≥

E. >, >, =, <

 

Questions: 6 – 10

Directions (Q. 6 – 10): Study the following information and answer the following  questions:

A, B, C, D, E, G, and I are seven friends who study in three different standards, namely 5th, 6th, and 7th, such that not less than two friends study in the same standard. Each friend has a different favourite subject, namely History, Civics, English, Marathi, Hindi, Maths and Economics also but not necessarily in the same order. A likes Maths and studies in the 5th standard with only one other friend who likes Marathi. I studies with two other friends. Both the friends who study with I like languages (here languages include only Hindi, Marathi and English). D studies in the 6th standard with only one person and does not like Civics. E studies with only one friend. The one who likes History does not study in the 5th or 6th standard. E does not like languages. C does not like English, Hindi or Civics.

 

Q. 6 Which combination represents E’s favourite subject and the standard in which he studies?

A. Civics and 7th

B. Economics and 5th

C. Civics and 6th

D. History and 7th

E. Economics and 7th

 

Q. 7 Which of the following is I’s favourite subject?

A. History

B. Civics

C. Marathi

D. Either English or Marathi

E. Either English or Hindi

 

Q. 8 Who among the following studies in the 7th standard?

A. G

B. C

C. E

D. D

E. Either D or B

 

Q. 9 Which of the following combinations is definitely correct?

A. I and Hindi

B. G and English

C. C and Marath

D. B and Hindi

E. E and Economics

 

Q. 10 Which of the following subjects does G like?

A. Either Maths or Marathi

B. Either Hindi or English

C. Either Hindi or Civics

D. Either Hindi or Marathi

E. Either Civics or Economics

 

Questions: 11 – 15

Directions (Q. 11 – 15): Study the information and answer the following questions.

In a certain code language ‘economics is not money’ is written as ‘ka la ho ga’, ‘demand and supply economics’ is written as ‘mo ta pa ka’, ‘money makes only part’ is written as ‘zi la ne ki’ and ‘demand makes supply economics’ is written as ‘zi mo ka ta’.

 

Q. 11 What is the code for ‘money’ in the given code language?

A. ga

B. mo

C. pa

D. ta

E. la

 

Q. 12 What is the code for ‘supply’ in the given code language?

A. Only ta

B. Only mo

C. Either pa or mo

D. Only pa

E. Either mo or ta

 

Q. 13 What may be the possible code for ‘demand only more’ in the given code language?

A. xi ne mo

B. mo zi ne

C. ki ne mo

D. mo zi ki

E. xi ka ta

 

Q. 14 What may be the possible code for ‘work and money’ in the given code language?

A. pa ga la

B. pa la tu

C. pa la tu

D. tu la ga

E. pa la ne

 

Q. 15 What is the code for ‘makes’ in the given code language?

A. mo

B. pa

C. ne

D. zi

E. ho

 

Questions: 16 – 20

Study the given information and answer the following questions:

When a word and number arrangement machine is given an input line of words and numbers, it arranges them following a particular rule. The following is an illustration of input an rearrangement (All the numbers are two-digit numbers). Input: 40 made butter 23 37 cookies salt extra 52 86 92 fell now 19

Step I: butter 19 40 made 23 37 cookies salt extra 52 86 92 fell now

Step II: cookies 23 butter 19 40 made 37 salt extra 52 86 92 fell now

Step III: extra 37 cookies 23 butter 19 40 made salt 52 86 92 fell now.

Step IV:  ell 40 extra 37 cookies 23 butter 19 made salt 52 86 92 now.

Step V: made 52 fell 40 extra 37 cookies 23 butter 19 salt 86 92 now.

Step VI: now 86 made 52 fell 40 extra 37 cookies 23 butter 19 salt 92

Step VII: Salt 92 now 86 made 52 fell 40 extra 37 cookies 23 butter 19

Step VII is the last step of the above arrangement as the intended arrangement is obtained.

As per the rules followed in the given steps, find out the appropriate steps for the given input.

Input: 32 proud girl beautiful 48 55 97 rich family 61 72 17 nice life.

 

Q. 16 How many steps will be required to complete the given input?

A. Five

B. Six

C. Seven

D. Eight

E. Nine

 

Q. 17 Which of the following is the third element from the left end of step VI?

A. beautiful

B. life

C. 61

D. nice

E. 17

 

Q. 18 Which of the following is step III of the given input?

A. proud 72 girl 48 family 32 beautiful 17 55 97 rich 61 nice life. proud 72 girl 48 family 32 beautiful 17 55 97 rich 61 nice life. proud 72 girl 48 family 32 beautiful 17 55 97 rich 61 nice life.

B. Life 55 girl 48 family 32 beautiful 17 proud 97 rich 61 72 nice

C. girl 48 family 32 beautiful 17 proud 55 97 rich 61 72 nice life

D. family 32 beautiful 17 proud girl 48 55 97 rich 61 72 nice life

E. girl 48 life 55 family 32 beautiful 17 proud 97 rich 61 72 nice

 

Q. 19 What is the position of ‘nice’ from the left end in the final step?

A. Fifth

B. Sixth

C. Seventh

D. Eighth

E. Ninth

 

Q. 20 Which element is third to the right of ‘family’ in Step V?

A. beautiful

B. 17

C. proud

D. 97

E. 32

 

Questions: 21 – 25

Directions (Q. 21 – 25): Read the information carefully and answer the following questions:

If A + B means A is the father of B.

If A – B means A is the sister of B

If A $ B means A is the wife of B.

If A % B means A is the mother of B.

If A | B means A is the son of B.

 

Q. 21 What should come in place of question mark to establish that J is brother of T in the expression? J | P % H ? T % L

A. –

B. |

C. $

D. Either | or –

E. Either + or |

 

Q. 22 Which of the given expressions indicates that M is daughter of D?

A. L % R $ D + T – M

B. L + R $ D + M – T

C. L % R % D + T | M

D. D + L $ R + M – T

E. L $ D | R % M | T

 

Q. 23 Which of the following options is true if the expression ‘I + T % J – L | K’ is definitely true?

A. L is daughter of T

B. K is son-in-law of I

C. I is grandmother of L

D. T is father of J

E. J is brother of L

 

Q. 24 Which of the following expressions is true if Y is son of X is definitely false?

A. W % L – T – Y | X

B. W + L – T – Y | X

C. X + L – T – Y | W

D. W $ X + L + Y + T

E. W % X + T – Y | L

 

Q. 25 What should come in place of question mark to establish that T is sister-in-law of Q in the expression. R % T – P ? Q + V

A. |

B. %

C. –

D. $

E. Either $ or –

 

Questions: 26 – 30

Directions (Q. 26 – 30): Study the following information and answer the questions given below:

Eight people – E, F, G, H, J, K, L and M – are sitting around a circular table, facing the centre. Each of them is of a different profession – Chartered Accountant, Columnist, Doctor, Engineer, Financial Analyst, Lawyer, Professor and Scientist, but not necessarily in the same order. F is sitting second to the left of K. The Scientist is an immediate neighbour of K. There are only three people between the Scientist and E. Only one person sits between the Engineer and E. The Columnist is on the immediate right of the Engineer. M is second to the right of K. H is the Scientist. G and J are immediate neighbours of each other. Neither G nor J is an Engineer. The Financial Analyst is on the immediate left of F. The Lawyer is second to the right of the Columnist. The Professor is an immediate neighbour of the Engineer. G is second to the right of the Chartered Accountant.

 

Q. 26 Who is sitting second to the right of E?

A. The Lawyer

B. G

C. The Engineer

D. F

E. K

 

Q. 27 Who among the following is the Professor?

A. F

B. L

C. M

D. K

E. J

 

Q. 28 Four of the following five are alike in a certain way based on the given arrangement and hence form a group. Which of the following does not belong to that group?

A. Chartered Accountant – H

B. M – Doctor

C. J – Engineer

D. Financial Analyst – L

E. Lawyer – K

 

Q. 29 What is the position of L with respect to the Scientist?

A. Third to the left

B. Second to the right

C. Second to the left

D. Third to the right

E. Immediate right

 

Q. 30 Which of the following statements is true according to the given arrangement?

A. The Lawyer is second to the left of the Doctor

B. E is an immediate neighbour of the Financial Analyst.

C. H sits exactly between F and the Financial Analyst.

D. Only four people sit between the Columnist and F

E. All of the given statements are true.

 

Questions: 31 – 35

In each of the questions below, two/three statements are given followed by conclusions/ group of conclusions numbered I and II. You have to assume all the statements to be true even if they seem to be at variance with the commonly known facts and then decide which of the given two conclusions logically follows from the information given in the statements.

Give answer as:

Option 1: if only conclusion I follows.

Option 2: if only conclusion II follows.

Option 3: if either conclusion I or conclusion II follows.

Option 4: if neither conclusion I nor conclusion II follows.

Option 5: if both conclusion I and II follow

 

Q. 31 Statements:

I. Some squares are circles.

II. No circle is a triangle.

III. No line is a square.

Conclusions:

I. All squares can never be triangles.

II. Some lines are circles.

A. Option 1

B. Option 2

C. Option 3

D. Option 4

E. Option 5

 

Q. 32 Statements:

Some squares are circles.

No circle is a triangle.

No line is a square.

Conclusions:

I. No triangle is a suqare.

II. No line is a circle

A. Option 1

B. Option 2

C. Option 3

D. Option 4

E. Option 5

 

Q. 33 Statements:

All songs are poems.

All poems are rhymes.

No rhyme is paragraph

Conclusions:

I. No Song is a paragraph.

II. No poem is a paragraph

A. Option 1

B. Option 2

C. Option 3

D. Option 4

E. Option 5

 

Q. 34 Statements:

All songs are poems.

All poems are rhymes.

No rhyme is paragraph

Conclusions:

I. All rhymes are poems.

II. All Songs are rhymes.

A. Option 1

B. Option 2

C. Option 3

D. Option 4

E. Option 5

 

Q. 35 Statements:

Some dews are drops.

All drops are stones

Conclusions:

I. At least some dews are stones.

II. At least some stones are drops

A. Option 1

B. Option 2

C. Option 3

D. Option 4

E. Option 5

 

Questions: 36 – 38

Each of the questions below consists of a question and two statements numbered I and II given below it. You have to decide whether the data given in the statements are sufficient to answer the questions. Read both the statements and give answer

1) if the data in statement I alone are sufficient to answer the question, while the data in statement II alone are not sufficient to answer the question.

2) if the data in statement II alone are sufficient to answer the question, while the data in statement I alone are not sufficient to answer the question.

3) if the data either in statement I alone or statement II alone are sufficient to answer the question.

4) if the data in statement I and II together are not sufficient to answer the question.

5) if the data in both statements I and II together are necessary to answer the question.

 

Q. 36 Seventeen people are standing in a straight line facing south. What is Bhavna’s position from the left end of the line?

I. Sandeep is standing second to the left of Sheetal. Only five people stand between Sheetal and the one who is standing at the extreme right end of the line. Four people stand between Sandeep and Bhavna.

II. Anita is standing fourth to the left of Sheetal. Less than three people are standing

between Bhavna and Anita.

A. 1

B. 2

C. 3

D. 4

E. 5

 

Q. 37 Five letters – A, E, G, N and R – are arranged from left to right according to certain conditions. Which letter is placed third?

I. G is placed second to the right of A. E is on the immediate right of G. There are only two letters between R and G.

II. N is exactly between A and G. Neither A nor G is at the extreme and of the arrangement.

A. 1

B. 2

C. 3

D. 4

E. 5

 

Q. 38 Six people – S, T, U, V, W and X – are sitting around a circular table facing the centre. What is T’s position with respect to X?

I. Only two people sit between U and W. X is second to the left of W. V and T are immediate neighbours of each other.

II. T is to the immediate right of V. There are only two people between T and S. X is an

immediate neighbour of S but not of V

A. 1

B. 2

C. 3

D. 4

E. 5

 

Questions: 39 – 40

Read the following information carefully and answer the given questions.

The convenience of online shopping is what I like best about it. Where else can you shop even at midnight wearing your night suit? You do not have to wait in a line or wait till the shop assistant is ready to help you with your purchases. It is a much better experience as compared to going to a retail store. – A consumer’s view

 

Q. 39 Which of the following can be a strong argument in favour of retail store owners?

A. Online shopping portals offer a great deal of discounts which retail stores offer only during the sale season

B. One can compare a variety of products online which cannot be done at retail stores

C. Many online shopping portals offer the ‘cash on delivery’ feature which is for those who are sceptical about online payments

D. Many consumers prefer shopping at retail stores which are nearer to their houses.

E. In online shopping the customer may be deceived as he cannot touch the product he is paying for

 

Q. 40 Which of the following can be inferred from the given information? (An inference is something that is not directly stated but can be inferred from the given information)

A. One can shop online only at night

B. Those who are not comfortable using computers can never enjoy the experience of online shopping

C. All retail stores provide shopping assistants to each and every customer.

D. The consumer whose view is presented has shopped at retail stores as well as online.

E. The consumer whose view is presented does not have any retail stores in her vicinity

 

Question 41

Read the following information carefully and answer the given question:

Many manufacturing companies are now shifting base to the rural areas of the country as there is a scarcity of space in urban areas. Analysts say that this shift will not have a huge impact on the prices of the products manufactured by these companies as only about 30% consumers live in urban areas.

 

Q. 41 Which of the following may be a consequence of the given information?

A. The prices of such products will decrease drastically in the urban areas.

B. People living in urban areas will not be allowed to work in such manufacturing companies.

C. These manufacturing companies had set-ups in the urban areas before shifting base.

D. Those who had already migrated to the urban areas will not shift back to rural areas

E. The number of people migrating from rural to urban areas in search of jobs may reduce

 

Question 42

Read the following information carefully and answer the given question:

‘Pets are not allowed in the park premises.’ – a notice put up at the park entrance by the authority that is responsible for maintenance of the park.

 

Q. 42 Which of the following can be an assumption according to the given information? (An assumption is something that is supposed or taken for granted)

A. At least some people who visit the park have pets.

B. This is the only park which does not allow pets

C. People who ignored this notice were fined

D. There are more than one entrance to the park

E. Many people have now stopped visiting the park

 

Questions: 43 – 45

Read the following information carefully and answer the given questions:

Despite repeated announcements that mobile phones were not allowed in the examination hall, three students were caught with their mobile phones.

(A) Mobile phones nowadays have a lot of features and it is easy to cheat with their help.

(B) The invigilator must immediately confiscate the mobile phones and ask the students to leave the exam hall immediately.

(C) Mobile phones are very expensive and leaving them in bags outside the exam hall is not safe.

(D) There have been incidents where students who left the exam hall early stole the mobile phones kept in the bags of the students who were writing the exam.

(E) The school authorities must ask the students to leave their phones in the custody of the invigilator before the exam in order to avoid thefts of mobile phones.

(F) None of the other students were carrying their phones in the exam hall

 

Q. 43 Which of the following among (A), (B), (C) and (D) may be a strong argument in favour of the three students who were caught with their mobile phones?

A. Only (A)

B. Both (A) and (B)

C. Both (C) and (D)

D. Only (C)

E. Both (B) and (D)

 

Q. 44 Which of the following among (A), (B), (E) and (F) may be the reason behind the school making such announcements before the exam?

A. Only (B)

B. Both (B) and (E)

C. Only (F)

D. Only (A)

E. Both (E) and (F)

 

Q. 45 Which of the following among (A), (B), (D) and (F) can be in immediate course of action for the invigilator?

A. Only (B)

B. Both (A) and (D)

C. Only (A)

D. Both (D) and (F)

E. Only (F)

 

Questions: 46 – 50

 

Q. 46 In the figure (I) what should be the next figure in the sequence from the five options (1),(2), (3),(4),(5) given in figure (II)

A. 1

B. 2

C. 3

D. 4

E. 5

 

Q. 47 In the figure (III) what should be the next figure in the sequence from the five options (1),(2), (3),(4),(5) given in figure (IV)

A. 1

B. 2

C. 3

D. 4

E. 5

 

Q. 48 In the figure (V) what should be the next figure in the sequence from the five options (1),(2), (3),(4),(5) given in figure (VI)

A. 1

B. 2

C. 3

D. 4

E. 5

 

Q. 49 In the figure (VI) what should be the next figure in the sequence from the five options (1),(2), (3),(4),(5) given in figure (VIII)

A. 1

B. 2

C. 3

D. 4

E. 5

 

Q. 50 In the figure (IX) what should be the next figure in the sequence from the five options (1),(2), (3),(4),(5) given in figure (X)

A. 1

B. 2

C. 3

D. 4

E. 5

 

Questions: 51 – 55

 

Q. 51 Study the following graph carefully to answer the questions that follow:

In which city is the difference between the cost of 1 kg of apple and the cost of one kg of guava the second lowest?

A. Jalandhar

B. Delhi

C. Chandigarh

D. Hoshiarpur

E. Ropar

 

Q. 52 The cost of 1 kg of guava in Jalandhar is approximately What per cent of the cost of 2 kg of grapes in Chandigarh?

A. 66

B. 24

C. 28

D. 34

E. 58

 

Q. 53 What total amount will Ram pay to the Shopkeeper for purchasing 3 kg of apples and 2 Kg. of guavas in Delhi?

A. ..

B. ..

C. ..

D. 620

E. 490

 

Q. 54 Ravinder had to purchase 45 kg of grapes from Hoshiarpur. The Shopkeeper gave him a discount of 4% per kg. What amount did he pay to the Shopkeeper after the discount?

A. 8208

B. 8104

C. 8340

D. 8550

E. 8410

 

Q. 55 What is the ratio of the cost of 1kg of apples from Ropar to the cost of 1kg. of grapes from Chandigarh?

A. 3:2

B. 2:3

C. 2²:3²

D. 4²:9²

E. 9²:4²

 

Questions: 56 – 60

 

Q. 56 Study the following graph carefully and answer the questions that follow

What was the difference between the number of students in University-1 in the year 2010 and the number of students in University-2 in the year 2012?

A. Zero

B. 5000

C. 15000

D. 10000

E. 1000

 

Q. 57 What is the sum of the number of students in University-1 in the year 2007 and the number of students in University-2 in the year 2011 together?

A. 50000

B. 55000

C. 45000

D. 57000

E. 40000

 

Q. 58 If 25% of the students in univeristy-2 in the year 2010 were females, what was the number of male students in University-2 in the same year?

A. 11250

B. 12350

C. 12500

D. 11500

E. 11750

 

Q. 59 What was the percent increase in the number of students in University-1 in the year 2011 as compared to the previous year?

A. 135

B. 15

C. 115

D. 25

E. 35

 

Q. 60 In which year was the difference between the number of students in University1 and the number of students in University-2 the highest?

A. 2008

B. 2009

C. 2010

D. 2011

E. 2012

 

Questions: 61 – 65

 

Q. 61 Study the graph carefully to answer the questions that follow

What is the total number of players participating in Hockey from all the five schools together?

A. 324

B. 288

C. 342

D. 284

E. 248

 

Q. 62 What is the ratio of the number of players participating in Basketball from School-1 to the number of players participating in Kho-Kho from School-3?

A. .

B. .

C. .

D. .

E. .

 

Q. 63 In which school is the number of players participating in Hockey and Basketball together the second highest?

A. School-1

B. School-2

C. School-3

D. School-4

E. School-5

 

Q. 64 The number of players participating in Kho-Kho from School-4 is what percent of the number of players participating in Hockey from School-2?

A. 42

B. 48

C. 36

D. 40

E. 60

 

Q. 65 25% of the number of players participating in Hockey from School-5 are females. What is the number of the Hockey players who are males in School-5?

A. 15

B. 18

C. 30

D. 21

E. 27

 

Questions: 66 – 70

 

Q. 66 Study the following bar-graph careflly and answer the following questions

What is Gita’s average earning over all the days togehter?

A. 285

B. 290

C. 320

D. 310

E. 315

 

Q. 67 What is the total amount earned by Rahul and Naveen together on Tuesday and Thursday together?

A. 1040

B. 1020

C. 980

D. 940

E. 860

 

Q. 68 Gita donated her earnings of Wednesday to Naveen. What was Naveen’s total earning on Wednesday after Gita’s donation?

A. 520

B. 550

C. 540

D. 560

E. 620

 

Q. 69 What is the difference between Rahul’s earning on Monday and Gita’s earning on Tuesday?

A. 40

B. 20

C. 50

D. 30

E. 10

 

Q. 70 What is the ratio of Naveen’s earning on Monday, Wednesday and Thursday?

A. 7 : 3 : 5

B. 8 : 6 : 5

C. 8 : 7 : 4

D. 9: 5 : 4

E. 9: 5 : 4

 

Questions: 71 – 75

 

 

Q. 71 Study the following pie-chart and answer the following questions.

What is the difference between the total number of employees in teaching and medical profession together and the number of employees in management profession?

A. 6770

B. 7700

C. 6700

D. 7770

E. 7670

 

Q. 72 In management profession three fourths of the number of employees are females. What is the number of male employees in management profession?

A. 1239

B. 1143

C. 1156

D. 1289

E. 1139

 

Q. 73 25% of employees from film production profession went on a strike. What is the number of employees from film production who did not participate in the strike?

A. 3271

B. 3819

C. 3948

D. 1273

E. 1246

 

Q. 74 What is the total number of employees in engineering profession and industries together?

A. 5698

B. 5884

C. 5687

D. 5896

E. 5487

 

Q. 75 In teaching profession if three – fifths of the teachers are not permenent, what is the number of permanent teachers in the teaching profession?

A. 1608

B. 1640

C. 1764

D. 1704

E. 1686

 

Questions: 76 – 80

 

Q. 76 Study the table carefully to answer the questions that follow:

What is the total amount of bill paid by Dev. in the month of June for all the four commodities?

A. 608

B. 763

C. 731

D. 683

E. 674

 

Q. 77 What is the average electricity bill paid by Manu over all the five months together?

A. 183

B. 149

C. 159

D. 178

E. 164

 

Q. 78 What is the difference between the mobile phone bill paid by Ravi in the month of May and the laundry bill paid by Dev in the month of March?

A. 180

B. 176

C. 190

D. 167

E. 196

 

Q. 79 In which months respectively did Manu pay the second highest mobile phone bill and the lowest electricity bill?

A. April and June

B. April and May

C. March and June

D. March and May

E. July and May

 

Q. 80 What is the ratio of the electricity bill paid by Manu in the month of April to the mobile phone bill paid by Ravi in the month of June?

A. 27 : 49

B. 27 : 65

C. 34:49

D. 135 : 184

E. 13 : 24

 

Questions: 81 – 85

 

Q. 81 Study the following table carefully and answer the questions that follow:

What is the distance travelled by the train from Surat to Nadiad Junction?

A. 176 km

B. 188 km

C. 183 km

D. 193 km

E. 159 km

 

Q. 82 How much time does the train take to reach Ahmedabad after departing from Anand Junction (including the halt time)?

A. 1 hr 59 min

B. 1 hr 17 min

C. 1 hr 47 min

D. 1 hr 45 min

E. 1 hr 15 min

 

Q. 83 What is the ratio of the number of passengers boarding from Vasai Road to that from Ahmedabad in the train?

A. 21:17

B. 13:9

C. 21:19

D. 15:13

E. 13:15

 

Q. 84 If the halt time (stopping time) of the train at Vadodara is decreased by 2 minutes and increased by 23 minutes at Ahmedabad, at what time will the train reach Bhuj?

A. 6.10 am

B. 6.01 pm

C. 6.05 am

D. 6.05 am

E. 6.07 pm

 

Q. 85 The distance between which two stations is the second lowest?

A. Nadiad Jn to Ahmedabad

B. Anand Jn to Nadiad Jn

C. Dadar to Vasai Road

D. Anand Jn to Vadodara

E. Vasai Road to Surat

 

Questions: 86 – 90

 

Q. 86 Study the table carefully to answer the questions that follow.

What is the difference between the maximum temperature of Ontario on 1st November and the minimum temperature of Bhuj on 1st January?

A. 3°C

B. 18°C

C. 15°C

D. 9°C

E. 11°C

 

Q. 87 In which month respectively is the maximum temperature of Kabul the second highest and the minimum temperature of Sydney the highest?

A. 1st October and 1st January

B. 1st October and 1st November

C. 1st December and 1st January

D. 1st September and 1st January

E. 1st December and 1st September

 

Q. 88 In which month (on 1st day) is the difference between maximum temperature

A. 1st September

B. 1st October

C. 1st November

D. 1st December

E. 1st January

 

Q. 89 What is the average maximum temperature of Beijing over all the months together?

A. 8.4°C

B. 9.6°C

C. 7.6°C

D. 9.2°C

E. 8.6°C

 

Q. 90 What is the ratio of the minimum temperature of Beijing on 1st September to the

maximum temperature of Ontario on 1st October?

A. 3:4

B. 3:5

C. 4:5

D. 1:5

E. 1:4

 

Questions: 91 – 95

 

Q. 91 Study the following pie-chart and table carefully to answer the questions that follow:

What is the difference between the number of diesel engine cars in State-2 and the number of petrol engine cars in State-4?

A. 159

B. 21

C. 28

D. 34

E. 161

 

Q. 92 The number of petrol engine cars in State-3 is what per cent more than the number of diesel engine cars in State-1?

A. 100

B. 200

C. 300

D. 125

E. 225

 

Q. 93 If 95% of diesel engine cars in State-3 are AC and the remaining cars are nonAC, what is the number of diesel engine cars in State-3 which are non-AC?

A. 75

B. 45

C. 95

D. 105

E. 35

 

Q. 94 What is the difference between the total number of cars in State-3 and the number of petrol engine cars in State-2?

A. 96

B. 106

C. 112

D. 102

E. 98

 

Q. 95 What is the average number of petrol engine cars in all the states together?

A. 86.75

B. 89.25

C. 89.75

D. 86.25

E. 88.75

 

Q. 96 A bag contains 7 blue balls and 5 yellow balls. If two balls are selected at random, what is the probability that none is yellow?

A. 5/33

B. 5/22

C. 7/22

D. 7/33

E. 7/66

 

Q. 97 A die is thrown twice. What is the probability of getting a sum 7 from both the throws?

A. 5/18

B. 1/18

C. 1/9

D. 1/6

E. 5/36

 

Questions: 98 – 100

Study the information carefully to answer these questions.

In a team, there are 240 members (males and females). Two-thirds of them are males. Fifteen per cent of males are graduates. Remaining males are nongraduates. Three-fourths of the females are graduates. Remaining females are nongraduates.

 

Q. 98 What is the difference between the number of females who are non-graduates and the number of males who are graduates?

A. 2

B. 24

C. 4

D. 116

E. 36

 

Q. 99 What is the sum of the number of females who are graduates and the number of males who are non-graduates?

A. 184

B. 96

C. 156

D. 84

E. 196

 

Q. 100 What is the ratio of the total number of males to the number of females who are nongraduates?

A. 6:1

B. 8:1

C. 8:3

D. 5:2

E. 7:2

 

Q. 101 Who was the Captain of Australian Cricket Team which currently (March 2013) visited India?

A. Michael Clarke

B. Shane Watson

C. Shane Warne

D. Michael Hussey

E. Ricky Ponting

 

Q. 102 Government, as part of the recapitalisation plan, infused capital in State Bank of India recently. Indicate the approximate capital infusion

A. Rs.500 Cr

B. Rs.1,000 Cr

C. Rs.1,500 Cr.

D. Rs.2,000 cr

E. Rs.3,000 Cr

 

Q. 103 The NRIs while investing in the equity of a company cannot buy more than a prescribed percentage of the paid-up capital of that company. What is the prescribed percentage?

A. 2%

B. 3%

C. 4%

D. 5%

E. 6%

 

Q. 104 Every year March 20 is celebrated as what day?

A. World Sparrow Day

B. International Women’s Day

C. World Cuckoo Day

D. International Children’s Day

E. International Mother’s Day

 

Q. 105 One of the famous Indian sportspersons released his/her autobiography ‘Playing to Win’ in November 2012. Name the sportsperson correctly

A. Saina Nehwal

B. Mary Kom

C. Yuvraj Singh

D. Sachin Tendulkar

E. Sushil Kumar

 

Q. 106 Which of the following terms is associated with the game of Cricket?

A. Bouncer

B. Love

C. Goal

D. Mid Court

E. Collision

 

Q. 107 Who is the author of the book Women of Vision?

A. Ravinder Singh

B. Preeti Shenoy

C. Amish Tripathi

D. Durjoy Dutta

E. Alam Srinivas

 

Q. 108 Prof. Muhammad Yunus, the recipient of the Nobel Peace Prize 2006, is the exponent of which of the following concepts in the filed of banking?

A. Core Banking

B. Micro Credit

C. Retail Banking

D. Real Time Gross Settlement

E. Internet Banking

 

Q. 109 Invisible export means export of

A. Services

B. Prohibited goods

C. Restricted goods

D. Good as per OGL list

E. Other than those given as options

 

Q. 110 The European Union has adopted which of the following as a common currency?

A. Dollar

B. Dinar

C. Yen

D. Euro

E. Peso

 

Q. 111 Banks is India are required to maintain a portion on of their demand and time liabilities with the Reserve Bank of India. This portion is called

A. Reverse Repo

B. Cash Reserve Ratio

C. Bank Deposit

D. Statutory Liquidity Ratio Statutory Liquidity Ratio Statutory Liquidity Ratio

E. Government Securities

 

Q. 112 Pre-shipment finance is provided by the banks only to

A. Credit Card Holders

B. Students aspiring for further studies

C. Brokers in equity market

D. Village Artisans

E. Exporters

 

Q. 113 Banking Ombudsman is appointed by

A. Government of India

B. State Governments

C. RBI

D. ECGC

E. Exim Bank

 

Q. 114 The Holidays for the Banks are declared as per

A. Reserve Bank Act

B. Banking Regulation Act

C. Negotiable Instruments Act

D. Securities and Exchange Board of India Act

E. Companies Act

 

Q. 115 Interest on Savings deposit nowadays is

A. Fixed by the RBI

B. Fixed by the respective Banks

C. Fixed by the depositors

D. Fixed as per the contract between Bank and the Consumer Court

E. Not paid by he Bank

 

Q. 116 Interest below which a bank is not expected to lend to customers is known as

A. Deposit Rate

B. Base rate

C. bank rate

D. Prime Lending Rate

E. Discount Rate

 

Q. 117 The customers by opening and investing in the Tax Saver Deposit Account Scheme in a Bank would get benefit under

A. Sales Tax

B. Customs Duty

C. Excise Duty

D. Professional Tax

E. Income Tax

 

Q. 118 In Banking business, when the borrowers avail a Term Loan, initially they are given a repayment holiday and this is referred as

A. Subsidy

B. Interest waiver

C. Re-phasing

D. Moratorium

E. Interest concession

 

Q. 119 One of the IT companies from India has become the first Indian Company to trade on NYSE Euronext London and Paris markets from February 2013 onwards. Which is this company?

A. Wipro Infotech Ltd.

B. L&T Infotech

C. HCL Technologies Ltd.

D. Infosys Technologies Ltd

E. Polaris Financial Technology Ltd

 

Q. 120 BSE (Bombay Stock Exchange), the oldest stock exchange in Asia, has joined hands with one more international index in February 2013. This association has resulted in change of name of BSE index. What is the change of name effected?

A. Dow Jones BSE Index

B. NASDAQ BSE Index

C. S&P BSE Index

D. Euronext BSE Index

E. Other than those given as options

 

Q. 121 A non-performing asset is

A. Money at call and short notice

B. An asset at cease to generate income

C. Cash balance in till

D. Cash balance with the RBI

E. Balance with other banks

 

Q. 122 RBI released its guidelines for entry of new banks in the private sector in the month of Febraury 2013. One of the norms is at least a prescribed percentage of branches of a new bank should be set up in unbanked rural centres with a population upto 9,999. What is the percentage of such branches prescribed in the norms?

A. 10%

B. 15%

C. 18%

D. 25%

E. Other than those given as options

 

Q. 123 The Union Budget for 2013-14 proposed by the Finance Minister on 28 th February 2013 announced introduction of a new variety of bonds by the Government. What is the name of these bonds?

A. Deep Discount Bonds

B. Zero Coupon bonds

C. Bullet Bonds

D. Inflation Indexed Bonds

E. Inflation Variable Bonds

 

Q. 124 Government usually classifies its expenditure in terms of planned and non-planned expenditure. Identify which is the correct definition of planned expenditure.

A. It represent the expenditure of all the State Governments

B. It represents the total expenditure or the Central Government.

C. It is the expenditure which is spent through centrally sponsored programmes and flagship schemes of the Government.

D. It reprensents the expenditure incurred on Defence

E. Other than those given as options

 

Q. 125 Which of the following organisations is made specifically responsible for empowering Micro, Small and Medium enterprises in India?

A. NABARD

B. RBI

C. SIDBI

D. ECGC

E. SEBI

 

Q. 126 ‘C’ in CPU denotes

A. Central

B. Common

C. Convenient

D. computer

E. Circuitry

 

Q. 127 A joystick is primarily used to/for

A. Print Text

B. Computer gaming

C. Enter text

D. Draw pictures

E. Control sound on the screen

 

Q. 128 Which is not a storage device?

A. CD

B. DVD

C. Floppy disk

D. Printer

E. Hard disk

 

Q. 129 Which of the following uses a handheld operating system?

A. A Supercomputer

B. A Personal computer

C. A Laptop

D. A Mainframe

E. A PDA

 

Q. 130 To display the contents of a folder in Windows Explorer you should

A. click on it

B. collapse it

C. name it

D. give it a password

E. rename it

 

Q. 131 The CPU comprises of Control, Memory, and —— units

A. Micro processor

B. Arithmetic/Logic

C. Output

D. ROM

E. Input

 

Q. 132 ……………. is the most important/powerful computer in a typical network

A. Desktop

B. Network client

C. Network server

D. Network station

E. Network switch

 

Q. 133 A (n) ……………. appearing on a web page opens another document when clicked.

A. Anchor

B. URL

C. Hyperlink

D. reference

E. heading

 

Q. 134 Which of the following refers to the rectangular area for displaying information and running programmes?

A. Desktop

B. Dialog box

C. Menu

D. Window

E. Icon

 

Q. 135 . ……………. is a Windows utility programme that locates and eliminates unnecessary fragments and rearranges files and unused disk space to optimise operations.

A. Backup

B. Disk Cleanup

C. Disk Cleanup

D. Restore

E. Disk restorer

 

Q. 136 Which of the following refers to too much electricity and may cause a voltage surge?

A. Anomaly

B. Shock

C. Spike

D. virus

E. Splash

 

Q. 137 The software that is used to create text-based documents are referred to as

A. DBMS

B. Suites

C. Spreadsheets

D. Presentation software

E. Word processors

 

Q. 138 . ……………. devices convert human-understandable data and programmes into a form that the computer can process.

A. Printing

B. Output

C. Solid state

D. Monitor

E. Input

 

Q. 139 Effective Selling Skills depend on the

A. Number of Languages known to the DSA

B. Data on marketing staf

C. Information regarding IT market

D. Knowledge of related markets

E. Ability to talk fast

 

Q. 140 A Direct Selling Agent (DSA) is required to be adept in

A. Surrogate marketing

B. Training skills

C. Communication skills

D. Market Research

E. OTC Marketing

 

Q. 141 Leads can be best sourced from

A. Foreign Customers

B. Yellow paages

C. Dictionary

D. List of vendors

E. Local supply chains

 

Q. 142 A successful marketing person requires one of the following qualities:

A. Empathy

B. Sympathy

C. Insistence

D. Aggressiveness

E. Pride

 

Q. 143 Innovation in marketing is the same as

A. Abbreviation

B. Communication

C. Creativity

D. Aspiration

E. Research work

 

Q. 144 Market segmentation can be resorted to by dividing the target group as per

A. Income levels of customers

B. age of the employees

C. Needs of the salespersons

D. Marketing skills of the employees

E. Size of the organisation

 

Q. 145 Post-sales activities include

A. Sales presentation

B. Customer’s feedback

C. Customer identification

D. Customer’s apathy

E. Product design

 

Q. 146 The ‘USP’ of a product denotes the

A. Usefulness of the product

B. Drawbacks of a Product

C. Main Functions

D. Number of allied products available

E. High selling features of a product

 

Q. 147 The competitive position of a company can be improved by

A. Increasing the selling price

B. Reducing the margin

C. Ignoring competitors

D. Increasing the cost price

E. Understanding and fulfilling customers’ needs

 

Q. 148 Value-added services means

A. Low-cost products

B. High-cost products

C. At-par services

D. Additional services for the same cost

E. Giving discounts

 

Q. 149 The target market for Debit Cards is

A. All existing account-holders

B. All agriculturists

C. All DSAs

D. All vendors

E. All Outsourced agents

 

Q. 150 A good Brand can be built up by way of

A. Customer Grievances

B. Break-down of IT support

C. Old age

D. Large number of products

E. Consistent offering of good services

 

Questions: 151 – 160

Read the following passage carefully and answer the questions given below it.

Certain words have been printed in bold to help you locate them while answering some of the questions. Financial Inclusion (FI) is an emerging priority for banks that have nowhere else to go to achieve business growth. The viability of FI Business is under Question, because while banks and their delivery partners continue to make investments, they haven’t seen commensurate returns. In markets like India, most programmes are focused on customer on boarding, an expensive process which people often find difficult to afford, involving issuance of smart cards to the customers. However, large scale customer acquisition hasn’t translated into large-scale business, with many accounts lying dormant and therefore yielding no return on the bank’s investment. For the same reason. Business Correspondent Agents, who constitute the primary channel for financial inclusion, are unable to pursue their activity as a full-time job. One major reason for this state of events is that the customer on-boarding process is often delayed after the submission of documents (required to validate the details of the concerned applicant) by the applicant and might take as long as two weeks. By this time initial enthusiasm of applicants fades away. Moreover, the delivery partners don’t have the knowledge and skill to propose anything other than the most basic financial products to the customer and hence do not serve their banks’ goal to expanding the offering in unbanked markets. Contrary to popular perception, the inclusion segment is not a singular impoverished, undifferentiated mass and it is important to navigate its diversity to identify the right target customers for various programmes. Rural markets do have their share of rich people who do not use banking services simply because they are inconvenient to access or have low perceived value. At the same time, urban markets, despite a high branch density, have multitude of low wageearners outside the financial  net. Moreover, the branch timings of banks rarely coincide with the off-work hours of the labour class. Creating affordability is crucial in tapping the unbanked market. No doubt pricing is a tool, but banks also need to be innovative in right-sizing their proposition to convince customers that they can derive big value even from small amounts. One way ‘of’ doing this is to show the target audience that a bank account is actually a lifestyle enabler, a convenient and safe means to send money to family or make a variety of purchases. Once banks succeed in hooking customers with this value proposition they must sustain their interest by introducing a simple and intuitive user application, ubiquitous access over mobile and other touch points, and adopting a banking mechanism which is not only secure but also reassuring to the customer. Technology is the most important element of financial inclusion strategy and an enabler of all others. The choice of technology is therefore a crucial decision, which could make or mar the agenda. Of the various selection criteria, cost is perhaps the most important. This certainly does not mean buying the cheapest package, but rather choosing that solution which by scaling transactions to huge volumes reduces per unit operating cost. An optimal mix of these strategies would no doubt offer an innovative means of expansion in the unbanked market.

 

Q. 151 Which of the following facts is true as per the passage?

A. People from rural areas have high perceived value of banking services

B. Cost is not a valid Criterion for technological package selection for financial inclusion initiatives

C. The inclusion segment is a singular impoverieshed, undifferentiated mass.

D. The branch timings of banks generally do not coincide with the off-work hours of the labour class in urban markets.

E. All the given statements are true.

 

Q. 152 According to the passage, for which of the following reasons do the delivery partners fail to serve their bank’s goal to expand in the unbanked markets?

(A) They do not have adequate client base to sell their financial products.

(B) They do not have adequate knowledge and skills to explain anything beyond basic financial products to the customers.

(C) They do not have the skills to operate advanced technological aids that are a prerequisite to tap the unbanked market.

A. Only (B)

B. Only (C)

C. All (A), (B) & (C)

D. Only (A)

E. Both (B) and (C)

 

Q. 153 According to the passage, for which of the following reasons is the viability of financial inclusion under question?

A. Banks always prefer the cheapest package (to cust cost) while making a choice of technology to be used.

B. The Business Correspondent Agents are highly demotiviated to pursue their activity as a full-time job.

C. The investments made by banks and their delivery partners are not yielding equal amounts of returns

D. Banks do not have adequate number of delivery partners required to tap the unbanked market.

E. Banks do not have adequate manpower to explore the diversity of the unbanked market and thereby identify the right target customers for various programmes.

 

Q. 154 In the passage, the author has specified which of the following characteristics of the customer on-boarding process?

A. In the passage, the author has specified which of the following characteristics of the customer on-boarding process?

B. It involves issuance of smart cards to the customers

C. It suffers from latency as it takes a long time after submission of documents by the customer

D. It is an expensive process which people find difficult to afford

E. All of the given characteristics have been specified

 

Q. 155 What did the author try to highlight in the passage?

(A) The ailing condition of financial inclusion business at present

(B) Strategies that may help banks expand in the unbanked market

(C) Role of government in modifying the existing financial-inclusion policies

A. Both (A) and (B)

B. All (A), (B) and (C)

C. Only (C)

D. Only (A)

E. Only (B)

 

Q. 156 According to the passage, which of the following ways may help banks sustain the interest of their customers after hooking them?

(A) Adoption of a banking mechanism which is not only secure but reassuring to the customers

(B) Increasing the number of delivery partners in rural market.

(C) Introduction of a simple and intuitive user application

A. Only (A)

B. Only (C)

C. Only (B)

D. All (A), (B) and (C)

E. Both (A) and (C)

 

Q. 157 Choose the word which is MOST SIMILAR in meaning to the word printed in bold as used in the passage.

Multitude

A. Impoverished

B. Handful

C. Acknowledged

D. Plenty

E. Solitued

 

Q. 158 Ubiquitous

A. Quintessential

B. Popular

C. Omnipresent

D. Simplified

E. Abnormal

 

Q. 159 Choose the word which is MOST OPPOSITE in meaning to the word printed in bold as used in the passage

Dormant

A. Emaciated

B. Pertinent

C. Cornered

D. Rejected

E. Active

 

Q. 160 Choose the word which is MOST OPPOSITE in meaning to the word printed in bold as used in the passage

Delayed

A. Perturbed

B. Popularised

C. Expedited

D. Stabilised

E. Repressed

 

Questions: 161 – 170

Read the following passage carefully and answer the questions given below it.

Certain words have been printed in bold to help you locate them while answering some of the questions. The evolution of Bring your Own Device (BYOD) trend has been as profound as it has been rapid. It represents the more visible sign that the boundaries

Between personal life and work life are blurring. The 9 am – 5 pm model of working solely from office has become archaic and increasingly people are working extended hours from a range of locations. At the very heart of this evolution is the ability to access enterprise networks from anywhere and anytime. The concept of cloud computing serves effectively to extend the office out of office. The much heralded benefit of BYOD is greater productivity. However, recent research has suggested that this is the greatest myth of BYOD and the reality is that BYOD in practice poses new challenges that may outweigh the benefits. A worldwide survey commissioned by Fortinet chose to look at attitudes towards BYOD and security from the user’s point of view instead of the IT Managers’. Specifically the survey was conducted in 15 territories on a group of graduate employees in their early twenties because they represent the fist Generation  to enter the workplace with an expectation of own device use. Moreover, they also represent tomorrow’s influencers and decision makers. The survey findings reveal that for financial organisations, the decision to embrace BYOD is extremely Dangerous. Larger organisations will have mature IT strategies and policies in place. But what about smaller financial business? They might not have such well developed Strategies to protect confidential data.  rucially, within younger employee group, 55% of the people share an expectation that they should be allowed to use their own devices in the workplace or for work purposes. With this expectation comes the very real risk that employees may consider contravening company policy banning the use of own devices. The threats posed by this level of subversion cannot be overstated. The survey casts doubt on the idea of BYOD leading to greater productivity by revealing the real reason people want to use their own devices. Only 26% of people in this age group cite efficiency as the reason they want to use their own devices, while 63% admit that the main reason  is so they have access to their favourite applications. But with personal applications so close to hand the risks to the business must surely include distraction and time wasting. To support this assumption 46% of people poled acknowledged time wasting as the greatest threat to the organisation, while 42% citing greater exposure to theft or loss of confidential data. Clearly, from a user perspective there is great deal of Contradiction surroundnig BYOD and there exists an undercurrent of selfishness where users expect to use their own devices, but mostly for personal interest. They recognise the risks to the organisation but are adamant that those risks are worth taking.

 

Q. 161 According to the passage, for which of the following reasons did Fortinet conduct the survey on a group of graduate employees in their early twenties?

(A) As this group represents the future decision makers.

(B) As this group represents the first generation who entered the workforce with a better understanding of sophisticated gadgets.

(C) As this group represents the first generation to enter the workplace expecting that they can use their own devices for work purpose

A. All (A), (B) and (c)

B. Only (C)

C. Both (A) and (C)

D. Only (A)

E. Only (B)

 

Q. 162 Which of the following is not true BYOD?

A. BYOD enables employees to access enterprise network from anywhere and anytime

B. Due to evolution of BYOD trend the 9 am- 5 pm model of working solely from office has become outdated.

C. Recent research has confirmed that BYOD boosts organisational productivity

D. The concept of cloud computing facilitates the BYOD trend

E. The concept of cloud computing facilitates the BYOD trend

 

Q. 163 According to the passage, why would the decision to embrace BYOD prove dangerous to smaller financial businesses?

A. Their employers have poor knowledge about their devices, which in turn poses a threat the confidential data of the organisation.

B. Their employees are more vulnerable to misplacement of devices.

C. They may lack mature IT strategies and policies required to protect confidential data.

D. They cannot afford to deal with damage liability issues of employee-owned devices.

E. Their employees have a tendency to change jobs frequently.

 

Q. 164 According to the passage, the expectation of Younger employees that they should be allowed to use their own devices in the workplace, entails which of the following risks?

A) Younger employees may deliberately transfer confidential data of their companies to rivals if they are not allowed to use their own devices for work purpose.

B) Younger employees may strongly feel like leaving the company if it prevents usage of their own device and join some other company that does not have such stringent policies.

C) Younger employees may consider flouting company policy prohibiting usage of their own devices in the workplace or for work purposes.

A. Only (C)

B. Only (B)

C. Both (A), (C)

D. Both (A), (C)

E. All (A), (B) and (C)

 

Q. 165 According to the findings of the survey conducted by Fortinet, why do majority of employees prefer using their own devices for work purpose?

A. As they often find that the devices provided by the company lack quality.

B. As they often find that the devices provided by the company lack quality.

C. As majority of them believe that output doubles when they use their own devices for work purpose

D. As handling data from their own devices reinforces their sence of responsibility

E. As it helps them create a brand of their own

 

Q. 166 What is/are the author’s main objective(s) in writing the passage?

(A) To break the myth that BYOD promotes employee efficiency and organisational productivity

(B) To suggest ways to control subversion across levels of corporate chain of command

(C) To throw light upon the fact that employees, even after knowing the risks involved, prefer to use their own devices for work purpose, mostly for personal benefits

A. Both (A) and (C)

B. All (A), (B) and (C)

C. Only (C)

D. Only (A)

E. Only (B)

 

Q. 167 Choose the word which is most SIMILAR in meaning to the word printed in bold as used in the passage.

Heralded

A. Suspected

B. Publicised

C. Dragged

D. Objective

E. Benefit

 

Q. 168 Choose the word which is most SIMILAR in meaning to the word printed in bold as used in the passage.

Outweigh

A. Control

B. Venture

C. Perish

D. Determine

E. Surpass

 

Q. 169 Choose the word which is most OPPOSITE in meaning to the word printed in bold as used in the passage.

Embrace

A. Contradict

B. Disobey

C. Curtail

D. Reject

E. Surpass

 

Q. 170 Choose the word which is most OPPOSITE in meaning to the word printed in bold as used in the passage.

Subversion

A. Compliance

B. Sanity

C. Popularity

D. Destabilisation

E. Clarity

 

Questions: 171 – 175

Read each sentence to find out whether there is any grammatical mistake/ error in it. The error, if any, will be in one part of the sentence. Mark the number of that part with error as your answer. If there is ‘No error’ , mark 5)

 

Q. 171 1) There cannot be any situation where

2) somebody makes money in an asset

3) located in India and does not pay tax

4) either to India or to the country of his origin.

5) No error

A. 1

B. 2

C. 3

D. 4

E. 5

 

Q. 172 1) India has entered a downward spiral

2) Where the organised, productive

3) and law abide sectors are subject to

4) savage amounts of multiple taxes.

5) No error

A. 1

B. 2

C. 3

D. 4

E. 5

 

Q. 173 1) The bank may have followed

2) an aggressive monetary tightening policy

3) but its stated aim of

4) curbing inflation have not been achieved.

5)No error

A. 1

B. 2

C. 3

D. 4

E. 5

 

Q. 174 1) Equal Opportunities for advancement

2) across the length and breadth

3) of an organisation will

4) keep many problems away.

5) No error

A. 1

B. 2

C. 3

D. 4

E. 5

 

Q. 175 1) A customised data science degree

2) is yet to become

3) a standard programme

4) to India’s Premier educational institutes.

5) No error

A. 1

B. 2

C. 3

D. 4

E. 5

 

Questions: 176 – 180

Each question below has two blanks, each blank indicating that something has been omitted. Choose the set of words for each blank that best fits the meaning of the sentence as a whole

 

Q. 176 When you want to digitalise a city ………. with millions, you don’t bet …………… the odds

A. proceeding, into

B. teeming, against

C. undergoing, adhere

D. dangling, for

E. falling, above

 

Q. 177 The numbers …………. by the legitimate online music service providers indicate that a growing number of users are ……… to buy music

A. morphed, ignoring

B. morphed, ignoring

C. morphed, ignoring

D. morphed, ignoring

E. morphed, ignoring

 

Q. 178 If India is ……….. on protecting its resources, international business appears equally ………….. to safeguard its profit.

A. dreaded, fragile

B. stubborn, weak

C. bent, determined

D. bent, determined

E. bent, determined

 

Q. 179 Brands ………. decision-simplicity strategies make full use of available information to ……… where consumers are on the path of decision making and direct them to the best market offers

A. diluting, divulge

B. diluting, divulge

C. diluting, divulge

D. pursuing, assess

E. employing, trust

 

Q. 180 Lack of financing options, ……….. with HR and technological ……….. make small and medium enterprises sector the most vulnerable component of our economy.

A. except, loophole

B. coupled, challenges

C. armed, benefits

D. registered, strategies

E. strengthened, facilities

 

Questions: 181 – 185

Rearrange the following six sentences (A), (B), (C), (D), (E) and (F) in the proper sequence to form a meaningful paragraph and then answer the questions given below .

(A) The group desired to enhance the learning experience in schools with an interactive digital medium that could be used within and outside the class-room.

(B) Then the teacher can act on the downloaded data rather than collect it from each and every student and thereby save his time and effort.

(C) Edutor, decided the group of engineers, all alumni of the Indian Institute of Technology, when they founded Edutor Technologies in August 2009.

(D) They can even take tests and submit them digitally using the same tablets and the teachers in turn can download, the tests using the company’s cloud services.

(E) With this desire they created a solution that digitises school text books and other learning material so that students no longer need to carry as many books to school and back as before, but can access their study material on their touchscreen tablets.

(F) A mechanic works on motors and an accountant has his computer. Likewise, if a student has to work on a machine of device, what should it be called?

 

Q. 181 Which of the following sentences should be the FIRST after rearrangement?

A. F

B. D

C. A

D. C

E. Which of the following sentences should be the FIRST after rearrangement? E

 

Q. 182 Which of the following sentences should be the THIRD after rearrangement?

A. A

B. B

C. D

D. E

E. F

 

Q. 183 Which of the following sentences should be the SIXTH (LAST) after rearrangement?

A. A

B. F

C. E

D. B

E. D

 

Q. 184 Which of the following sentences should be the FOURTH after rearrangement?

A. A

B. F

C. E

D. B

E. C

 

Q. 185 Which of the following sentences should be the FIFTH after rearrangement?

A. A

B. D

C. C

D. E

E. F

 

Questions: 186 – 195

In the following passage there are blanks, each of which has been numbered. These numbers are printed below the passage and against each, five words/ phrases are suggested, one of which fits the blank appropriately. Find out the appropriate word/ phrase in each case. There is a considerable amount of research about the factors that make a company innovate. So is it possible to create an environment (186) to innovation? This is a particularly pertinent (187) for India today. Massive problems in health, education etc (188) be solved using a conventional Approach but (189) creative and innovative solutions that can ensure radical change and (190). There are several factors in India’s (191). Few countries have the rich diversity that India or its large, young population (192). While these (193) innovation policy interventions certain additional steps are also required. These include (194) investment in research and development by (195) the government and the private sector, easy transfer of technolgy from the academic world etc. To fulfil its promise of being prosperious and to be at the forefront, India must be innovative.

 

Q. 186 1) stimuli

2) conducive

3) incentive

4) facilitated

5) impetus

A. 1

B. 2

C. 3

D. 4

E. 5

 

Q. 187 1) objective

2) controversy

3) doubt

4) question

5) inference

A. 1

B. 2

C. 3

D. 4

E. 5

 

Q. 188 1) cannot

2) possibly

3) should

4) never

5) must

A. 1

B. 2

C. 3

D. 4

E. 5

 

Q. 189 1) necesary

2) apply

3) need

4) consider

5) requires

A. 1

B. 2

C. 3

D. 4

E. 5

 

Q. 190 1) quantity

2) advantages

3) increase

4) chaos

5) growth

A. 1

B. 2

C. 3

D. 4

E. 5

 

Q. 191 1) challenges

2) praises

3) favour

4) leverage

5) esteem

A. 1

B. 2

C. 3

D. 4

E. 5

 

Q. 192 1) blessed

2) enjoys

3) endows

4) prevails

5) occurs

A. 1

B. 2

C. 3

D. 4

E. 5

 

Q. 193 1) aid

2) jeopardise

3) promotes

4) endure

5) cater

A. 1

B. 2

C. 3

D. 4

E. 5

 

Q. 194 1) acute

2) utilising

3) restricting

4) inspiring

5) increased

A. 1

B. 2

C. 3

D. 4

E. 5

 

Q. 195 1) both

2) besides

3) combining

4) participating

5) also

A. 1

B. 2

C. 3

D. 4

E. 5

 

Questions: 196 – 200

In each of the following questions four words are given of which two words are most nearly the same or opposite in meaning. Find the two words which are most nearly the same or opposite in meaning and indicate the number of the correct letter combination, by darkening the appropriate oval in your answer sheet.

 

Q. 196 (A) consent

(B) nascent

(C) emerging

(D) Insecure

A. A C

B. B D

C. B C

D. A D

E. A B

 

Q. 197 (A) elated

(B) eccentric

(C) explicit

(D) abnormal

A. A B

B. B D

C. A C

D. A D

E. D C

 

Q. 198 (A) abundance

(B) incomparable

(C) Projection

(D) plethora

A. A C

B. A B

C. C D

D. B D

E. A D

 

Q. 199 (A) purposefully

(B) inaccurately

(C) inadvertently

(D) unchangeably

A. A C

B. A B

C. B C

D. B D

E. A D

 

Q. 200 (A) germane

(B) generate

(C) reliable

(D) irrelevant

A. B D

B. B C

C. A B

D. C D

E. A D

 

 

Answer Sheet
Question 1 2 3 4 5 6 7 8 9 10
Answer B D D E C C A A C B
Question 11 12 13 14 15 16 17 18 19 20
Answer E E A B D C D C A B
Question 21 22 23 24 25 26 27 28 29 30
Answer A B B D D B D C B A
Question 31 32 33 34 35 36 37 38 39 40
Answer A D E B E E C B E D
Question 41 42 43 44 45 46 47 48 49 50
Answer E A C D A D C B B C
Question 51 52 53 54 55 56 57 58 59 60
Answer B D C A C B E A D E
Question 61 62 63 64 65 66 67 68 69 70
Answer B C B D E B D C A E
Question 71 72 73 74 75 76 77 78 79 80
Answer C E B D A C C A D A
Question 81 82 83 84 85 86 87 88 89 90
Answer C E A B C E A C E B
Question 91 92 93 94 95 96 97 98 99 100
Answer B A D E B C D C E B
Question 101 102 103 104 105 106 107 108 109 110
Answer A E D A A A E B A D
Question 111 112 113 114 115 116 117 118 119 120
Answer B E C C B B E E D C
Question 121 122 123 124 125 126 127 128 129 130
Answer B D D C C A B D E A
Question 131 132 133 134 135 136 137 138 139 140
Answer B C C D C C E E D C
Question 141 142 143 144 145 146 147 148 149 150
Answer E A C A A E E D A E
Question 151 152 153 154 155 156 157 158 159 160
Answer D A C E A D C C E D
Question 161 162 163 164 165 166 167 168 169 170
Answer C C C A B A B E D A
Question 171 172 173 174 175 176 177 178 179 180
Answer B C D E D B E C D B
Question 181 182 183 184 185 186 187 188 189 190
Answer A A D C B B D A C E
Question 191 192 193 194 195 196 197 198 199 200
Answer C B A E A C B E A E

 

SBI PO 2011 Mains Previous Year Paper

SBI PO 2011 Mains

Section

Questions

Marks

Logical Reasoning

50 Questions (1 – 50)

50

Quantitative Aptitude

20 Questions (51 – 70)

20

Data Interpretation

30 Questions (71 – 100)

30

General Knowledge

50 Questions (101 – 150)

50

English

50 Questions (151 – 200)

50

Questions: 1 – 3

Directions (Q 1-3) : Study the following information carefully and answer the given questions.

Four of the following five are alike in a certain way and hence form a group. Which is one that does not belong to the group?

 

Q. 1 The word which doesn’t belong in the group is –

A. Break

B. Change

C. Split

D. Divide

E. Separate

 

Q. 2 The word which doesn’t belong in the group is –

A. Train

B. Instruct

C. Educate

D. Advocate

E. Coach

 

Q. 3 The word which doesn’t belong in the group is –

A. Extend

B. Higher

C. Upward

D. Rise

E. Ascend

 

Q. 4 Which of the following symbols should replace question mark in the given expression in order to make the expressions ‘A>D’ and ‘F>C’ definitely true?

A>B≥C?D≤E=F

A. >

B. <

C. ≤

D. =

E. Either = or ≥

 

Q. 5 Which of the following expressions is definitely true if the given expressions ‘RQ’ are definitely true?

A. P>Q=R≤T<S

B. S>T≥R>Q<P

C. Q>R≤T>P≥S

D. S>T≥R>Q>P

E. None of these

 

Q. 6 Read the following information carefully and answer the question which follows:

‘A x B’ means ‘A is the father of B’

‘A + B’ means ‘A is the daughter of B’

‘A ÷ B’ means ‘A is the son of B’

‘A – B’ means ‘A is the sister of B’

What will come in place of question mark to establish that P is the son-in law of S in the following expression?

P x Q + R – T ? S

A. +

B. x

C. –

D. ÷

E. Either + or ÷

 

Questions: 7 – 12

Directions (Q 7-12) :

Study the following information to answer the given questions :

Twelve people are sitting in two parallel rows containing six people each such that they are equidistant from each other. In row 1: P, Q, R, S, T and V are seated and all of them are facing South. In row 2: A, B, C, D, E and F are seated and all of them are facing North. Therefore, in the given seating arrangement, each member seated in a row faces another member of the other row. S sits third to the right of Q. Either S or W sits at an extreme end of the line. The one who faces Q sits second to the right of E. Two people sit between B and F. Neither B nor F sits at an extreme end of the line. The immediate neighbor of B faces the person who sits third to the left of P. R and T are immediate neighbors. C sits second to the left of A. T does not face the immediate neighbor of D.

 

Q. 7 Who amongst the following sit at the extreme ends of the rows?

A. S, D

B. Q, A

C. V, C

D. P, D

E. Q, F

 

Q. 8 Who amongst the following faces S?

A. A

B. B

C. C

D. D

E. F

 

Q. 9 How many persons are seated between V and R?

A. One

B. Two

C. Three

D. Four

E. None of these

 

Q. 10 P is related to A in the same way as S is related to B based on the given arrangement. Which f the following is T related to following the same pattern?

A. C

B. D

C. E

D. F

E. Cannot be determined

 

Q. 11 Which of the following is true regarding T?

A. F faces T

B. V is an immediate neighbor of T

C. F faces the one who is second to the right of T

D. T sits at one of the extreme ends of the line

E. Q sits second to the right of T

 

Q. 12 Four of the following five are alike in a certain way based on the given arrangement and so form a group. Which is the one that does not belong to that group?

A. A-T

B. B-T

C. F-P

D. C-V

E. E-Q

 

Questions: 13 – 17

Directions (Q 13-17) :

Each of the questions below consists of a question and two statements numbered I and II given below it. You have to decide whether the data provided in the statements are sufficient to answer the question. Read both the statements and give answer.

 

Q. 13 Which bag amongst P, Q, R, S and T is the heaviest?

I. Bag Q is heavier than R and S. Bag T is heavier than only bag P.

II. Only three bags are lighter than R. the weight of bag Q is 50 kg, which is 2 kg more than bag R.

A. The data in statement I alone is sufficient to answer the question, while the data in statement II alone is not sufficient to answer the question.

B. The data in statement II alone is sufficient to answer the question, while the data in statement I alone is not sufficient to answer the question

C. The data in either statement I or II alone are sufficient to answer the question

D. The data in both statements I and II are not sufficient to answer the question

E. The data in both statements I and II together are necessary to answer the question

 

Q. 14 Are all five friends – A, B, C, D and E – who are seated around a circular table facing the centre?

I. A sits to the left of B. B faces the centre. D and E are immediate neighbors of each other. C sits second to the right of E.

II. D sits second to right of C. C faces the centre. Both E and A are immediate neighbors of D. B sits second to the right of A.

A. The data in statement I alone is sufficient to answer the question, while the data in statement II alone is not sufficient to answer the question.

B. The data in statement II alone is sufficient to answer the question, while the data in statement I alone is not sufficient to answer the question

C. The data in either statements I or II alone are sufficient to answer the question

D. The data in both statements I and II are not sufficient to answer the question

E. The data in both statements I and II together are necessary to answer the question

 

Q. 15 In a college, five different subjects viz Physics, Chemistry, Botany, Zoology and Mathematics are taught on five different days of the same week, starting from Monday and ending on Friday. Is Chemistry taught on Wednesday?

I. Two subjects are taught between Zoology and Mathematics. Mathematics is taught before Zoology. Chemistry is taught on the day immediately next to the day when Physics is taught. Botany is not taught on Friday.

II. Three lectures are scheduled between the lectures of Botany and Zoology. Mathematics is taught immediately before Physics.

A. The data in statement I alone is sufficient to answer the question, while the data in statement II alone is not sufficient to answer the question.

B. The data in statement II alone is sufficient to answer the question, while the data in statement I alone is not sufficient to answer the question

C. The data in either statements I or II alone are sufficient to answer the question

D. The data in both statements I and II are not sufficient to answer the question

E. The data in both statements I and II together are necessary to answer the question

 

Q. 16 Is it 9 o’clock now?

I. After half an hour, the minute and the hour hands of the clock will make an angle of exactly 90° with each other.

II. Exactly 15 minutes ago, the hour and the minute hands of the clock coincided with each other.

A. The data in statement I alone is sufficient to answer the question, while the data in statement II alone is not sufficient to answer the question.

B. The data in statement II alone is sufficient to answer the question, while the data in statement I alone is not sufficient to answer the question

C. The data in either statements I or II alone are sufficient to answer the question

D. The data in both statements I and II are not sufficient to answer the question

E. The data in both statements I and II together are necessary to answer the question

 

Q. 17 Is F granddaughter of B?

I. B is the father of M. M is the sister of T. T is the mother of F.

II. S is the son of F. V is the daughter of F. R is the brother of T.

A. The data in statement I alone is sufficient to answer the question, while the data in statement II alone is not sufficient to answer the question.

B. The data in statement II alone is sufficient to answer the question, while the data in statement I alone is not sufficient to answer the question

C. The data in either statements I or II alone are sufficient to answer the question

D. The data in both statements I and II are not sufficient to answer the question

E. The data in both statements I and II together are necessary to answer the question

 

Questions: 18 – 20

Directions (Q 18 -20) :

Study the following information to answer the given questions:

A word and number arrangement machine when given an input line of words and numbers rearranges them following a particular rule in each step. The following is an illustration of input and rearrangement. (All the numbers are two-digit numbers and are arranged as per some logic based on the value of the number) Input: win 56 32 93 bat for 46 him 28 11 give chance

Step I : 93 56 32 bat for 46 him 28 11 give chance win

Step II : 11 93 56 32 bat for 46 28 give chance win him

Step III : 56 11 93 32 bat for 46 28 chance win him give

Step IV : 28 56 11 93 32 bat 46 chance win him give for

Step V : 46 28 56 11 93 32 bat win him five for chance bat

Step VI is the last step of the arrangement in the above input. As per the rules followed in the above steps, find out in each of the following questions the appropriate steps for the given input.

Input for the questions :

Input : fun 89 at the 28 16 base camp 35 53 here 68

(All numbers given in the arrangement are 2 digit numbers)

 

Q. 18 Which of the following would be step II?

A. 89 fun at 28 base camp 35 53 here 68 the

B. 35 53 28 68 16 89 the here fun camp base at

C. 16 89 at fun 28 camp base 35 53 68 the here

D. 53 28 68 16 89 35 the here fun camp base at

E. None of these

 

Q. 19 Which word/number would be at seventh position from the left in step IV?

A. base

B. at

C. 35

D. The

E. 53

 

Q. 20 Which step number would be the following output?

53 28 68 16 89 at 35 the here fun camp base

A. There will be no such step

B. III

C. II

D. V

E. IV

 

Questions: 21 – 27

Directions (Q 21-27) :

Study the following information carefully and answer the given questions.

Eight colleagues A, B, C, D, E, F, G and H are sitting around a circular table facing the centre but not necessarily in the same order. Each one of them holds a different post – Manager, Company Secretary, Chairman, President, Vice President, Group Leader, Financial Advisor and Managing Director. A sits third to the right of the Managing Director. Only two people sit between the Managing Director and H. The Vice President and the Company Secretary are immediate neighbors. Neither A nor H is a Vice President or a Company Secretary. The Vice President is not an immediate neighbor of the Managing Director. The Manager sits second to the left of E. E is not an immediate neighbor of H. The Manager is an immediate neighbor of both the Group Leader and the Financial Advisor. The Financial Advisor sits third to the right of B. B is not the Vice President. C sits on the immediate right of the Chairman. A is not the Chairman. F is not an immediate neighbor of A. G is not an immediate neighbor of the Manager.

 

Q. 21 Who amongst the following sits third to the left of E?

A. Manager

B. G

C. A

D. Financial advisor

E. B

 

Q. 22 Four of the following five are alike in a certain way based on the given arrangement and thus form a group. Which is the one that does not belong to that group?

A. F – Chairman

B. G – President

C. D – Manager

D. A – Financial Advisor

E. B – Managing Director

 

Q. 23 Who among the following is the President of the company?

A. A

B. C

C. H

D. G

E. D

 

Q. 24 Which of the following is true with respect to the given seating arrangement?

A. The Group Leader of the company is an immediate neighbor of the Vice President

B. G sits second to the right of D

C. The Group Leader and the Company Secretary are immediate neighbors

D. The Chairman of the company sits to the immediate left of the Managing Director

E. The Group Leader sits second to the left of D

 

Q. 25 Which of the following post does B hold in the company?

A. Chairman

B. Manager

C. Company Secretary

D. Vice President

E. Financial Advisor

 

Q. 26 Who among the following sits exactly between the Managing Director and H?

A. H and the Chairman

B. B and G

C. The Chairman and C

D. F and C

E. E and the Group Leader

 

Q. 27 Who among the following is the Group Leader?

A. C

B. F

C. G

D. H

E. A

 

Questions: 28 – 31

Directions (Q 28-31) :

Read the following information and the sentences (A), (B), (C), (D) and (E) given below and answer the questions that follow:

A host of foreign companies are in talks with the Indian government for selling B150, a tough, short-haul plane ideal for connectivity of smaller towns which is lacking in India at present.

(A) B150 planes have not only low operating costs than competing planes like Cezana but also a much better track record in terms of safety and efficiency

(B) The profit margin of road transport operators in the smaller towns connected by B150 planes has been reduced substantially as a majority of people prefer air transport over other means of transport

(C) Smaller towns, at present, are better connected by roads and railways as compared to flight services

(D) B150 planes are capable of operating in sectors where large airlines cannot fly due to challenging conditions such as mist, short runways, etc. Such planes can also double as cargo planes and charter flights for the rich and the elite.

(E) B150 planes need to operate in the existing airports which are suited in bigger cities only and are poorly connected to the smaller cities.

 

Q. 28 Which of the statements (A), (B), (C), (D) and (E) can be inferred from the facts/information given in the statement? (An interference is something which is not directly stated but can be inferred from the given facts)

A. Only A

B. Only B

C. Only C

D. Both B and D

E. Only E

 

Q. 29 Which of the statements (A), (B), (C), (D) and (E) mentioned above would weaken the offer made by the foreign companies for selling B150 planes to Indian government?

A. A

B. B

C. C

D. D

E. E

 

Q. 30 Which of the statements (A), (B), (C), (D) and (E) mentioned above represents a possible consequence of the success B150 planes in smaller cities?

A. A

B. B

C. C

D. D

E. E

 

Q. 31 Which of the statements (A), (B), (C), (D) and (E) would favor the foreign companies; bid to sell B150 planes in India?

A. Only A

B. Only B

C. Both B and C

D. Both A and D

E. Both E and C

 

Questions: 32 – 37

Directions (Q 32-37) :

Study the following information to answer the given questions :

In a certain code, ‘always create new ideas’ is written as ‘ba ri sha gi’. ‘ideas and new thoughts’ is written as ‘fa gi ma ri’, ‘create thoughts and insights’ is written as ‘ma job a fa’ and ‘new and better solutions is written as ‘ki ri to fa’.

 

Q. 32 What is the code for ‘ideas’?

A. sha

B. ba

C. gi

D. ma

E. Cannot be determined

 

Q. 33 What does ‘fa’ stand for?

A. Thoughts

B. Insights

C. New

D. And

E. Solutions

 

Q. 34 ‘fa lo ba’ could be a code for which of the following?

A. Thoughts and action

B. Create and innovate

C. Ideas and thoughts

D. Create new solutions

E. Always better ideas

 

Q. 35 What is the code for ‘new’?

A. ki

B. ri

C. to

D. fa

E. ba

 

Q. 36 Which of the following may represent ‘insights always better’?

A. jo ki to

B. ki to ri

C. sha jo ri

D. to sha jo

E. sha to ba

 

Q. 37 What is the code for ‘thoughts’?

A. ma

B. fa

C. ba

D. jo

E. Either jo or fa

 

Q. 38 Read the following information carefully and answer the questions that follow.

All existing and upcoming hotels within a 5 km radius of national parks and sanctuaries in India will have to pay 30% of their annual turnover as tax to the government.

Which of the following statements can be inferred from the facts/information given in the above statements?

A. The tax collected from the hotels will be used for the betterment of these national parks and sanctuaries

B. Hotels which are sponsored by the government will not have to pay any tax even if these are located within 5 km radius of such wildlife hotspots

C. The ecosystem of the national parks and sanctuaries is adversely affected even if the hotels are located outside the 5 km radius

D. Government allows construction of hotels within 5 km radius of national parks and sanctuaries

E. Such a step is taken by the environment ministry to boost eco-tourism and perk up revenue collection of State governments

 

Q. 39 Read the following paragraph and answer the questions that follow.

Tenants’ associations have demanded a ‘vacancy tax’ on all vacant and unsold flats in Mumbai. Which of the following would support the demand made by the tenants’ association?

A. House owners too have demanded for laws that make it easier to evict tenants who default on rent payment

B. Such a tax law would be difficult to implement as it would be difficult to record the number of vacant flats in the city

C. People with surplus money buy many houses and rent these out while many consumers cannot afford even their first house

D. The number of vacant flats in Mumbai is lower than other metros such as Delhi and Chennai where vacancy tax already exists

E. Such a tax would compel the house owners to sell/rent properties which in turn would control hoarding and skyrocketing property prices

 

Questions: 40 – 45

Directions (Q 40-45) :

In each question below are given two/three statements followed by two conclusions numbered I and II. You have to take the given statements to be true even if they seem to be at variance with commonly known facts. Read all the conclusions and then decide which of the given conclusions logically follows from the given statements, disregarding commonly known facts.

 

Q. 40 Statements :

All rings are circles

All squares are rings

No ellipse is a circle

Conclusions :

I. Some rings being ellipses is a possibility

II. At least some circles are squares

A. Only conclusion I follows

B. Only conclusion II follows

C. Either conclusion I or conclusion II follows

D. Neither conclusion I nor conclusion II follows

E. Both conclusion I and conclusion II follows

 

Q. 41 Statements :

No house is an apartment

Some bungalows are apartments

Conclusions :

I. No house is a bungalow

II. All bungalow are houses

A. Only conclusion I follows

B. Only conclusion II follows

C. Either conclusion I or conclusion II follows

D. Neither conclusion I nor conclusion II follows

E. Both conclusion I and conclusion II follows

 

Q. 42 Statements :

Some gases are liquids

All liquids are water

Conclusions :

I. All gases being water is a possibility

II. All such gases which are not water can never be liquids

A. Only conclusion I follows

B. Only conclusion II follows

C. Either conclusion I or conclusion II follows

D. Neither conclusion I nor conclusion II follows

E. Both conclusion I and conclusion II follows

 

Q. 43 Statements :

All minutes are seconds

All seconds are hours

No second is a day

Conclusions :

I. No day is an hour

II. At least some hours are minutes

A. Only conclusion I follows

B. Only conclusion II follows

C. Either conclusion I or conclusion II follows

D. Neither conclusion I nor conclusion II follows

E. Both conclusion I and conclusion II follows

 

Q. 44 Statements :

Some teachers are professors

Some lecturers are teachers

Conclusions :

I. All teachers as well as professors being lecturers is a possibility

II. All those teachers who are lecturers are also professors

A. Only conclusion I follows

B. Only conclusion II follows

C. Either conclusion I or conclusion II follows

D. Neither conclusion I nor conclusion II follows

E. Both conclusion I and conclusion II follows

 

Q. 45 Statements :

Some teachers are professors

Some lecturers are teachers

Conclusions :

I. No professor is a lecturer

II. All lecturers being professors is a possibility

A. Only conclusion I follows

B. Only conclusion II follows

C. Either conclusion I or conclusion II follows

D. Neither conclusion I nor conclusion II follows

E. Both conclusion I and conclusion II follows

 

Q. 46 Which one of the five Answer figures should come after the Problem Figures on the left, if the sequence were continued?

A. 1

B. 2

C. 3

D. 4

E. 5

 

Q. 47 Which one of the five Answer figures should come after the Problem Figures on the left, if the sequence were continued?

A. 1

B. 2

C. 3

D. 4

E. 5

 

Q. 48 Which one of the five Answer figures should come after the Problem Figures on the left, if the sequence were continued?

A. 1

B. 2

C. 3

D. 4

E. 5

 

Q. 49 Which one of the five Answer figures should come after the Problem Figures on the left, if the sequence were continued?

A. 1

B. 2

C. 3

D. 4

E. 5

 

Q. 50 Which one of the five Answer figures should come after the Problem Figures on the left, if the sequence were continued?

A. 1

B. 2

C. 3

D. 4

E. 5

 

Questions: 51 – 55

Directions (Q 51-55) :

What will come in place of question mark (?) in the following questions?

 

Q. 51  32.05% of 259.99 = ?

A. 92

B. 88

C. 8

D. 90

E. 83

 

Q. 52  1/8 of 2/3 of 3/5 of 1715 = ?

A. 80

B. 85

C. 90

D. 95

E. 75

 

Q. 53  25.05 x 123.95 + 388.999 x 15.001 = ?

A. 900

B. 8950

C. 8935

D. 8975

E. 8995

 

Q. 54  561 ÷ 35.05 x 19.99 = ?

A. 320

B. 330

C. 315

D. 325

E. 335

 

Q. 55  (15.01)² x √730 = ?

A. 6125

B. 6225

C. 6200

D. 6075

E. 6250

 

Questions: 56 – 60

Directions (Q 56-60) :

In each of these questions, a number series is given. In each series, only one number is wrong. Find out the wrong number.

 

Q. 56  3601 3602 1803 604 154 36 12

A. 3602

B. 1803

C. 604

D. 154

E. 36

 

Q. 57  4 12 42 196 1005 6066 42511

A. 12

B. 42

C. 1005

D. 196

E. 6066

 

Q. 58  2 8 12 20 30 42 56

A. 8

B. 42

C. 30

D. 20

E. 12

 

Q. 59  32 16 24 65 210 945 5197.5

A. 945

B. 16

C. 24

D. 210

E. 65

 

Q. 60  7 13 25 49 97 194 385

A. 13

B. 49

C. 97

D. 194

E. 25

 

Q. 61 Mr X invested a certain amount in Debt and Equity Funds in the ratio of 4:5. At the end of one year, he earned a total dividend of 30% on his investment. After one year, he reinvested the amount including the dividend in the ratio of 6:7 in Debt and Equity Funds. If the amount reinvested in Equity Funds was ₹94,500, what was the original amount invested in Equity Funds?

A. ₹75,000

B. ₹81,000

C. ₹60,000

D. ₹65,000

E. None of these

 

Q. 62 The age of the father is 30 years more than the son’s age. Ten years hence, the father’s age will become three times the son’s age that time. What is the son’s present age in years?

A. Eight

B. Seven

C. Five

D. Cannot be determined

E. None of these

 

Q. 63 If the length of a rectangular field is increased by 20% and the breadth is reduced by 20%, the area of the rectangle will be 192 m². What is the area of the original rectangle?

A. 184 m²

B. 196 m²

C. 204 m²

D. 225 m²

E. None of these

 

Q. 64 The product of one-third of a number and 150% of another number is what percent of the product of the original numbers?

A. 80%

B. 50%

C. 75%

D. 120%

E. None of these

 

Q. 65 Inside a square plot, a circular garden is developed which exactly fits in the square plot and the diameter of the garden is equal to the side of the square plot which is 28 meters. What is the area of the space left out in the square plot after developing the garden?

A. 98 m²

B. 146 m²

C. 84 m²

D. 168 m²

E. None of these

 

Q. 66 Amit and Sujit together can complete an assignment of data entry in five days. Sujit’s speed is 80% of Amit’s speed and the total key depressions in the assignment are 5,76,000. What is Amit’s speed in key depressions per hour if they work for 8 hours a day?

A. 4800

B. 6400

C. 8000

D. 7200

E. None of these

 

Q. 67 Out of five girls and three boys, four children are to be randomly selected for a quiz

contest. What is the probability that all the selected children are girls?

A. 1/14

B. 1/7

C. 5/17

D. 2/17

E. None of these

 

Q. 68 Profit earned by an organization is distributed among officers and clerks in the ratio of 5:3. If the number of officers is 45 and the number of clerks is 80 and the amount received by each officer is ₹25,000, what was the total amount of profit earned?

A. ₹22 lakh

B. ₹18.25 lakh

C. ₹18 lakh

D. ₹23.25 lakh

E. None of these

 

Q. 69 A shopkeeper labeled the price of his articles so as to earn a profit of 30% on the cost price. He then sold the articles by offering a discount of 10% on the labeled price. What is the actual percent profit earned in the deal?

A. 18%

B. 15%

C. 20%

D. Cannot be determined

E. None of these

 

Q. 70 Mr Shamin’s salary increases every year by 10% in June. If there is no other increase or reduction in the salary and his salary in June 2011 was ₹22,385, what was his salary in June 2009?

A. ₹18,650

B. ₹18,000

C. ₹19,250

D. ₹18,500

E. None of these

 

Questions: 71 – 75

Directions (Q 71-75) :

In each of these questions, one question is given followed by data in three statements I, II and III. You have to study the question and the data in statements and decide the question can be answered with data in which of the statements and mark your answer accordingly.

 

Q. 71 What is the rate of interest pcpa?

Statements :

I. The difference between the compound interest and simple interest earned in two years on the amount invested is ₹100.

II. The amount becomes ₹19,500 in three years on simple interest

III. The simple interest accrued in two years on the same amount at the same rate of interest is ₹3,000.

A. Only I and II

B. Only I and III

C. Only II and III

D. Only I and either II or III

E. None of these

 

Q. 72 What is the speed of the train in kmph?

Statements :

I. The train crosses an ‘x’ meter-long platform in ‘n’ seconds

II. The length of the train is ‘y’ meters

III. The train crosses a signal pole in ‘m’ seconds

A. Any two of the three

B. Only II and III

C. Only I and III

D. All I, II and III

E. Question cannot be answered even with information in all three statements

 

Q. 73 How many students passed in first class?

Statements :

I. 85% of the students who appeared in examination have passed either in first class or in second class or in pass class

II. 750 students have passed in second class

III. The number of students who passed in pass class is 28% of those passed in second class

A. All I, II and III

B. Only I and III

C. Only II and III

D. Question cannot be answered even with information in all three statements

E. None of these

 

Q. 74 What is the amount invested in Scheme ‘B’?

Statements :

I. The amounts invested in Schemes ‘A’ and ‘B’ are in the ratio 2:3

II. The amount invested in Scheme ‘A’ is 40% of the total amount invested

III. The amount invested in Scheme ‘A’ is ₹45,000

A. Only I and II

B. Only I and III

C. Only II and III

D. All I, II and III

E. Only III and either I or II

 

Q. 75 What is the cost of flooring a rectangular hall?

Statements :

I. The length of the rectangle is 6 meters

II. The breadth of the rectangle is two-thirds of its length

III. The cost of flooring the area of 100 cm² is ₹45

A. Only I and III

B. Only II and III

C. All I, II and III

D. Question cannot be answered even with data in all three statements

E. None of these

 

Questions: 76 – 83

Directions (Q 76-83) :

Study the following graph carefully to answer these questions.

 

Q. 76 If the profit earned in 2006 by Company B was ₹8,12,500, what was the total income of the company in that year?

A. ₹12,50,000

B. ₹20,62,500

C. ₹16,50,000

D. ₹18,25,000

E. None of these

 

Q. 77 If the amount invested by the two companies in 2005 was equal, what was the ratio of the total income of the Company A to that of B in 2005?

A. 31:33

B. 33:31

C. 34:31

D. 14:11

E. None of these

 

Q. 78 If the total amount invested by the two companies in 2009 was ₹27 lakh, while the amount invested by Company B was 50% of the amount invested by Company A, what was the total profit earned by the two companies together?

A. ₹21.15 lakh

B. ₹20.70 lakh

C. ₹18.70 lakh

D. ₹20.15 lakh

E. None of these

 

Q. 79 If the income of Company A in 2007 and that in 2008 were equal and the amount invested in 2007 was ₹12 lakh, what was the amount invested in 2008?

A. ₹10,87,500

B. ₹10,85,700

C. ₹12,45,000

D. ₹12,85,000

E. None of these

 

Q. 80 If the amount of profit earned by Company A in 2006 was ₹10.15 lakh, what was the total investement?

A. ₹13.8 lakh

B. ₹14.9 lakh

C. ₹15.4 lakh

D. ₹14.2 lakh

E. None of these

 

Q. 81 If the amount invested by Company B in 2004 is ₹12 lakh and the income of 2004 is equal to the investment in 2005, what is the amount of profit earned in 2005 by Company B?

A. ₹6.6 lakh

B. ₹18.6 lakh

C. ₹10.23 lakh

D. ₹9.6 lakh

E. None of these

 

Q. 82 If the investments of Company A in 2007 and 2008 were equal, what is the difference between the profits earned in the two years if the income in 2008 was ₹24 lakh?

A. ₹2.25 lakh

B. ₹3.6 lakh

C. ₹1.8 lakh

D. ₹2.6 lakh

E. None of these

 

Q. 83 If each of the companies A and B invested ₹25 lakh in 2010, what was the average profit earned by the two companies?

A. ₹18 lakh

B. ₹22.5 lakh

C. ₹17.5 lakh

D. ₹20 lakh

E. None of these

 

Questions: 84 – 90

Directions (Q 84-90) :

Study the following table carefully and answer the questions which follow.

 

Q. 84 From State B, which post had the highest percentage of candidates short listed?

A. V

B. IV

C. VI

D. II

E. None of these

 

Q. 85 What is the average number of candidates (approximately) found eligible for Post III from all states?

A. 6700

B. 6200

C. 4200

D. 4500

E. 5500

 

Q. 86 What is the overall percentage (rounded off to one digit after decimal) of candidates shortlisted over the total number of candidates eligible for Post I from all the States together?

A. 9.5%

B. 12.5%

C. 7.2%

D. 6.5%

E. None of these

 

Q. 87 What is the ratio of the total number of candidates short-listed for all the posts together from State E to that from State G?

A. 307:369

B. 73:79

C. 6:5

D. 9:7

E. None of these

 

Q. 88 The total number of candidates found eligible for Post I from all states together is approximately what percent of the total number of candidates found eligible for Post VI from all States together?

A. 45%

B. 50%

C. 60%

D. 55%

E. 65%

 

Q. 89 Which state had the lowest percentage of candidates short listed with respect to candidate eligible for Post IV?

A. G

B. F

C. E

D. C

E. None of these

 

Q. 90 What is the ratio of the total number of candidates short listed for Post V to that for post VI from all states together?

A. 6:7

B. 55:96

C. 165:278

D. 16:25

E. None of these

 

Questions: 91 – 95

Directions (Q 91-95) :

These questions are based on the following data. Study it carefully and answer the questions that follow.

In s school having 400 students, boys and girls are in the ratio of 3:5. The students speak Hindi, English or both the languages. 12% of the boys speak only Hindi. 22% of the girls speak only English. 24% of the total students speak only Hindi and the number of boys speaking both the languages is six times the number of boys speaking only Hindi.

 

Q. 91 How many boys speak Hindi?

A. 18

B. 126

C. 108

D. 26

E. None of these

 

Q. 92 How many girls speak only Hindi?

A. 55

B. 117

C. 96

D. 78

E. None of these

 

Q. 93 How many students speak English?

A. 304

B. 79

C. 225

D. 117

E. None of these

 

Q. 94 The number of girls speaking only Hindi is what percent of the total number of students speaking only Hindi?

A. 38.2%

B. 71.8%

C. 31.2%

D. 78%

E. None of these

 

Q. 95 What is the ratio of the number of boys to the number of girls speaking both the

Languages?

A. 23:25

B. 12:25

C. 12:13

D. 25:13

E. None of these

 

Q. 96 The area of the circle is 616 cm². What is the area of the rectangle? ( ● or ‘dot’ indicates centre of the circle)

A. 784 cm²

B. 196 cm²

C. 392 cm²

D. Cannot be determined

E. None of these

 

Q. 97 What is the average female population in million?

A. 8.32

B. 8.86

C. 8.68

D. 9.12

E. None of these

 

Q. 98 What is the percent rise in production in 2007 from 2006? (Round off to two digits after decimal)

A. 28.18%

B. 18.18%

C. 16.28%

D. 26.18%

E. None of these

 

Q. 99 Out of a total 550 students, how many students did not prefer Maths or Economics?

A. 462

B. 154

C. 196

D. 396

E. None of these

 

Q. 100 What is the difference (in ₹ lakh) between the average salary and the lowest salary?

A. 4.2

B. 2.65

C. 3.65

D. 4.06

E. None of these

 

Q. 101  A major Public Sector Bank raised interest rates on loans by 25 basis points – was newsin some major financial newspapers recently. This means the bank has raised interest by 25 basis points of

A. Savings Bank Interest Rate

B. Base Lending Rate

C. Repo Rates

D. Present Rates on Deposits

E. Discounted Rates of Interest

 

Q. 102 The Finance Minister of India in one of his press conferences said that inflationary pressure is likely to continue following recent increase in rates of some commodities in international markets. Which of the following commodities was he referring to as it gets frequent increase at international levels and disturbs our Home Economy substantially?

A. Gold and Silver

B. Petroleum Products

C. Tea and Coffee

D. Sugar

E. Jute and Jute products

 

Q. 103 Immediately prior to change in the measure of Food Inflation, which of the following indexes was being used for measuring it?

A. Wholesale Price Index

B. Consumer Price Index

C. Interest Rates offered by banks on deposits

D. Sensex and Nifty

E. None of these

 

Q. 104 What is the full form of ‘FINO’, a term we see frequently in financial newspapers?

A. Financial investment Network and Operations

B. Farmer’s Investment in National Organisation

C. Farmers Inclusion News and Operations

D. Financial Inclusion Network and Operations

E. None of these

 

Q. 105 Which of the following is/are commodity/ies on which subsidy is given to farmers/poor in India?

(A) Fertilizer

(B) Kerosene

(C) LPG

A. Only (A)

B. Only (B)

C. Only (C)

D. Only (A) and (C)

E. All (A), (B) and (C)

 

Q. 106 ‘Bhagyam Oilfields’, which were recently in news, are located in which of the following states in India?

A. Rajasthan

B. Gujarat

C. Assam

D. Maharashtra

E. Karnataka

 

Q. 107 The Finance Minister of India recently decided to review the position of ‘Bad Debts’ in priority sector. Which of the following is/are the reason(s) owing to which FM took this Decision?

(A) Bad Debt in agriculture is still rising substantially every year despite special treatment to it

(B) Bad Debt in other areas of priority sector is almost nil now

(C) Govt. is planning to disburse another installment of ₹1,60,000 crore, which will bring ‘Bad Debts’ almost at ‘Nil level’ and no special treatment will be needed in next fiscal year

A. Only (A)

B. Only (B)

C. Only (C)

D. Both (A) and (B)

E. All (A), (B) and (C)

 

Q. 108 Which amongst the following countries is NOT a member of the TAPI pipeline project, which was in news recently?

A. India

B. Afghanistan

C. Portugal

D. Pakistan

E. Turkmenistan

 

Q. 109 An agreement on Arms Reduction popularly known as ‘START’ is one of the major issues in the way of smooth relations between which of the following two countries?

A. India-Pakistan

B. India-China

C. China-Japan

D. USA-Russia

E. Russia-Iran

 

Q. 110 The International WWW Conference – 2011 was organized in which of the following places in March 2011?

A. London

B. Paris

C. Vienna

D. Manila

E. Hyderabad

 

Q. 111 Which of the following international agencies/organizations had initiated an ‘International Convention for the Protection of All Persons from Enforced Disappearance’, which came into effect recently?

A. World Health Organisation (WHO)

B. International Court of Justice

C. International Maritime Organisation

D. International Atomic Energy Agency (IAEA)

E. United Nations Organisation (UNO)

 

Q. 112 ‘BRIC’ which was the organization of 4 nations, namely Brazil, Russia, India and China, has now become ‘BRICS’. Which is the fifth nation in it?

A. South Korea

B. Sri Lanka

C. Singapore

D. Spain

E. South Africa

 

Q. 113 The process of acquisition of agricultural land and its compensation to farmers recently came up as a major issue in which of the following states? (It also created law and order problem in the state)

A. Haryana

B. Gujarat

C. Uttar Pradesh

D. Rajasthan

E. Punjab

 

Q. 114 What does the letter ‘L’ denote in the term ‘LAF’ as referred every now and then in relation to monetary policy of the RBI?

A. Liquidity

B. Liability

C. Leveraged

D. Longitudinal

E. Linear

 

Q. 115 BP Plc, which was in news recently, is a major international company in the field of

A. Heavy Machinery

B. Oil exploration and processing

C. Atomic Energy

D. Information Technology

E. None of these

 

Q. 116 Who amongst the following has been appointed the new coach of the Indian Cricket team for a period of two years?

A. Stuart Law

B. Allan Border

C. Gary Kirsten

D. Duncan Fletcher

E. Allan Donald

 

Q. 117 Why was Kanishtha Dhankar’s name in news recently?

A. She was crowned Femina Miss India 2011

B. She was adjudged Best Actress 2010 by Film fare

C. She is the new captain of India’s woman cricket team

D. She has become the new Dy Minister of Textile in place of Dayanidhi Maran

E. None of these

 

Q. 118 K Balachander, who was awarded prestigious Dada Saheb Phalke Award 2010, is basically a/an

A. Actor

B. Director

C. Photographer

D. Music Director

E. Choreographer

 

Q. 119 Banks and other financial institutions in India are required to maintain a certain amount of liquid assets like cash, precious metals and other short-term securities as a reserve all the time. In Banking World, this is known as

A. CRR

B. Fixed Asset

C. SLR

D. PLR

E. None of these

 

Q. 120 Which of the following space agencies recently launched three satellites and placed them into Polar Sun Synchronous Orbit successfully?

A. NASA

B. ROSCOSMOS

C. cnsa

D. ISRO

E. JAXA

 

Q. 121 As per the decision taken by the Govt. of India recently, “Bamboo is a liberated item now”. What does it really mean?

(A) Bamboo, henceforth, will be treated as a minor forest produce only

(B) Now villagers are free to sell bamboo in their community forest areas

(C) Gram Sabha has been given right to issue transport passes to legally transport the produce

A. Only (A)

B. Only (B)

C. Only (C)

D. All (A), (B) and (C)

E. None of these

 

Q. 122 Union Cabinet recently passed a ₹7000-crore project to clean up which of the following rivers in India?

A. Narmada

B. Ravi

C. Ganga

D. Chambal

E. Krishna

 

Q. 123 As per estimates of the Planning Commission of India, the target growth rate of 12th Five Year Plan is set at

A. 7 to 8%

B. 8.0 to 8.5%

C. 9 to 9.5%

D. 10 to 10.5%

E. 11%

 

Q. 124 Goodluck Jonathan was recently re-elected the President of

A. South Africa

B. Nigeria

C. Kosovo

D. Haiti

E. None of these

 

Q. 125 Which of the following schemes has NOT been launched by the Ministry of Rural

Development, Govt. of India?

A. National Old Age Pension Scheme

B. National Maternity Benefit Scheme

C. National Family Benefit Scheme

D. Mid-day Meal Scheme

E. All are launched by Ministry of Rural Development

 

Q. 126 Where are you likely to find an embedded operating system?

A. On a desktop operating system

B. On a networked PC

C. On a networked Server

D. On a PDA

E. On a mainframe

 

Q. 127 An online discussion group that allows direct “live” communication is known as

A. Webcrawler

B. Chat group

C. Regional service provider

D. Hyperlink

E. E-mail

 

Q. 128 Which of the following is a program that uses a variety of different approaches to identify and eliminate spam?

A. Directory search

B. Anti-spam program

C. Web server

D. Web storefront creation package

E. Virus

 

Q. 129 Connection or link to other documents or Web Pages that contain related information is called

A. Dial-up

B. Electronic commerce

C. Hyperlink

D. E-cash

E. Domain name

 

Q. 130 Which of the following is a programming language for creating special programs like applets?

A. Java

B. Cable

C. Domain name

D. Net

E. COBOL

 

Q. 131 The system unit

A. Coordinates input and output devices

B. Is the container that houses electronic components

C. Is a combination of hardware and software

D. Controls and manipulates data

E. Does the arithmetic operations

 

Q. 132 System software

A. Allows the user to diagnose and troubleshoot the device

B. Is a programming language

C. Is part of a productivity suite

D. Is an optional form of software

E. Helps the computer manage internal resources

 

Q. 133 Computer and communication technology, such as communication links to the Internet, that provide help and understanding to the end user is known as

A. Presentation file

B. Information technology

C. Program

D. Worksheet file

E. FTP

 

Q. 134 Which of the following is contained on chips connected to the system board and is a holding area for data instructions and information? (processed data waiting to be output

to secondary storage)

A. Program

B. Mouse

C. Internet

D. Memory

E. Modem

 

Q. 135 Microsoft’s Messenger allows users to

A. Bypass a browser to surf the Web

B. Create a blog

C. Communicate via direct live communication

D. Identify and eliminate spam

E. Make graphic presentations

 

Q. 136 Portable computer, also known as a laptop computer, weighing between 4 and 10 pounds is called

A. General-purpose application

B. Internet

C. Scanner

D. Printer

E. Notebook computer

 

Q. 137 The main circuit-board of the system unit is the

A. Computer program

B. Control unit

C. Motherboard

D. RAM

E. None of these

 

Q. 138 Which of the following is billionth of a second?

A. Gigabyte

B. Terabyte

C. Nanosecond

D. Microsecond

E. Terasecond

 

Q. 139 Online Marketing is the function of which of the following?

A. Purchase section

B. Production Department

C. IT Department

D. Designs Section

E. A collective function of all staff

 

Q. 140 Customization is useful for

A. Designing customer specific products

B. Call centers

C. Publicity

D. Motivating the staff

E. Cold calls

 

Q. 141 The key challenge to market-driven strategy is

A. Selling maximum products

B. Employing maximum DSAs

C. Delivering superior value to customers

D. Being rigid to changes

E. Adopting short-term vision

 

Q. 142 Effective selling skills depends on

A. Size of the sales team

B. Age of the sales team

C. Peer strength

D. Knowledge level of the sales team

E. Educational level of the sales team

 

Q. 143 Generation of sales leads can be improved by

A. Being very talkative

B. Increasing personal and professional contacts

C. Being passive

D. Engaging Recovery Agents

E. Product Designs

 

Q. 144 A Market Plan is

A. Performance Appraisal of marketing staff

B. Company Prospectus

C. Documented marketing strategies

D. Business targets

E. Call centre

 

Q. 145 Marketing channels mean

A. Delivery objects

B. Sales targets

C. Delivery outlets

D. Delivery boys

E. Sales teams

 

Q. 146 Social Marketing means

A. Share market prices

B. Marketing by the entire society

C. Internet Marketing

D. Marketing for a social cause

E. Society bylaws

 

Q. 147 Service Marketing is the same as

A. Internet Marketing

B. Telemarketing

C. Internal Marketing

D. Relationship Marketing

E. Transaction Marketing

 

Q. 148 Market-driven strategies include

A. Identifying problems

B. Planning marketing tactics of peers

C. Positioning the Organisation and its brands in the marketplace

D. Internal marketing

E. Selling old products

 

Q. 149 Innovation in marketing is same as

A. Motivation

B. Perspiration

C. Aspiration

D. Creativity

E. Team work

 

Q. 150 Personal Loans can be canvassed among

A. Salaried persons

B. Pensioners

C. Foreign Nationals

D. NRI customers

E. Non-customers

 

Questions: 151 – 160

Directions (Q 151-160) :

Read the following passage carefully and answer the questions given below.

Certain words/phrases have been given in quotes to help you locate them while answering some of the questions.

Jagir Singh has sold red onions at a market in south Delhi everyday for the past half-century. Perched on an upturned crate, wrapped tight against the chill air, he offers pyaz, a staple for much Indian cooking, for 60 rupees a kilo, the most he can remember. Business is brisk but most customers pick up only a small handful of onions. That is just as well. Wholesale supplies are “tight”, he says, and the quality is poor. As India’s economy grows by some 9% a year, food prices are soaring. In late December, the commerce ministry judged that food inflation had reached 18.3%, with pricey vegetables mostly to blame. Officials have made some attempts to “temper” the rise in the past month – scrapping import taxes for onions, banning their export and ordering low-priced sales at government-run shops. But there is no quick fix. Heavy rain in the west of India brought a roten harvest. Vegetables from farther afield – including a politically sensitive delivery from a neighbouring country – are costly to move on India’s crowded, pot-holed roads. Few refrigerated lorries and poor logistics mean that much of each harvest is wasted. Newspapers allege hoarders are “cashing in”. The biggest problems are structural. Food producers, hampered by land restrictions, “archaic” retail networks and bad infrastructure, fail to meet extra demand from consumers. It was estimated in October that a 39% rise in income per person in the previous five years might have created an extra 220 million regular consumers of milk, eggs, meat and fish. Supplies have not kept up with this potential demand. The broader inflation rate may be a less eye-watering problem than the onions suggest. The central bank has lifted interest rates steadily in the past year and is expected to do so again later this month. Headline inflation fell to 7.5% in November, down by just over a percentage point from October, though it is still above the central bank’s forecast of 5.5% for March.

 

Q. 151 What is responsible for the increased demand of certain food items amongst consumers?

A. There has been an increase in the wholesale supplies of this food stuff

B. The vegetables in the market are highly priced

C. There has been an increase in the incomes of people

D. There is a lack of availability of vegetables in the market

E. There has been a surge in population

 

Q. 152 Which of the following is not true in the context of the passage?

A. The overall inflation is not as bad as the food inflation in India

B. Help from other countries to counter food inflation has proved to be quite encouraging

C. Government is banning the export of certain types of vegetables in order to check food inflation

D. Highly priced vegetables are mostly responsible for the increased food inflation

E. All the above statements are true

 

Q. 153 Which of the following is/are the reason/s for increase in food/vegetable prices?

(A) Bad weather

(B) Land restrictions

(C) Poor infrastructure for storage and transportations

A. Only (B)

B. Only (A) and (C)

C. Only (A) and (B)

D. Only (B) and (C)

E. All (A), (B) and (C)

 

Q. 154 Which of the following is/are TRUE in the context of the passage?

(A) Interest rates are being raised in India

(B) India is witnessing a steady economic growth

(C) It has been proven that the food inflation is mainly because of hoarding

A. All (A), (B) and (C)

B. Only (A) and (B)

C. Only (C)

D. Only (B) and (C)

E. None is true

 

Q. 155 Which of the following is possibly the most appropriate title for the passage?

A. Food Inflation in India

B. Onions and Vendors

C. Food Deficit Worldwide

D. Food Imports in India

E. Benefits to Indian Consumers

 

Q. 156 What can be said about the sale of onions at present as given in the passage?

A. Vegetables vendors are unwilling to sell onions

B. People are not buying as much as they used to

C. The sale of onions has picked up and is unprecedented

D. People are buying more onions than they used to

E. None of these

 

Q. 157 The usage of the phrase “cashing in” in the passage can possible mean

A. Profiting

B. Running away

C. Paying money

D. Bailing out

E. Buffering

 

Q. 158 Which of the following is most similar in meaning to the word “tight” as used in the passage?

A. Firm

B. Loose

C. Limited

D. Taut

E. Tense

 

Q. 159 Which of the following is most similar in meaning to the word “temper” as used in the passage?

A. Displeasure

B. Anger

C. Rage

D. Harness

E. Control

 

Q. 160 Which of the following is most opposite in meaning of the word “archaic” as used in the passage?

A. Simple

B. Straightforward

C. Modern

D. Lively

E. Ancient

 

Questions: 161 – 165

Directions (Q161 – 165) :

Which of the phrases given below each sentence in the options should replace the phrase given in quotes in the sentence to make it grammatically correct? If the sentence is correct as it is then choose option 5 as the correct answer.

 

Q. 161 Seeing that there was an ongoing sale in one of her favourite stores, Seeta “made a bee line for” it immediately after entering the mall

A. Made a bee’s line for

B. Make bees lined to

C. Made a bee line to

D. Make bee line to

E. No correction required

 

Q. 162 Sharon made it to work in the “nicks of time”, or else she would have missed the meeting

A. Nick of time

B. Nicked time

C. Nick of timeliness

D. Nick and time

E. No correction required

 

Q. 163 Varun was “on cloud nine” after having stood first in his class

A. In ninth cloud

B. On nine clouds

C. A cloudy nine

D. Cloud on nine

E. No correction required

 

Q. 164 Vithal had a habit of “pass the buck” when it came to important issues at work

A. Pass to bucking

B. Passing buck

C. Passing the buck

D. Pass buck

E. No correction required

 

Q. 165 Puneet “racked his brains” and tried to find an answer to a tricky question given in the paper but couldn’t find one

A. Rake his brain

B. Racked his brains

C. Racked brains

D. Raked brain

E. No correction required

 

Questions: 166 – 170

Directions (Q 166-170) :

Each question has two blanks, each blank indicating that something has been omitted. Find out which option can be used to fill up the blank in the sentence in the same sequence to make it meaningfully complete.

 

Q. 166 The water transport project on the west coast is ________ to get a shot in the arm with a new plan in which the Road Development Corporation will build the infrastructure and ________ a private party to operate the service.

A. scheduled, let

B. verge, permit

C. set, sanctions

D. slated, allow

E. bound, task

 

Q. 167 As the weekend finally rolled around, the city folk were only ________ happy to settle down and laugh their cares ________.

A. just, fair

B. too, away

C. extremely, off

D. very, up

E. so, on

 

Q. 168 The flood of brilliant ideas has not only ________ us, but has also encouraged us to ________ the last date for submission of entries.

A. overwhelmed, extend

B. enjoyed, stretch

C. dismayed, decide

D. scared, scrap

E. happy, boundary

 

Q. 169 ________ about prolonged power cuts in urban areas, the authorities have decided to ________ over to more reliable and eco-friendly systems to run its pumps.

A. worried, shift

B. frantic, move

C. troubled, jump

D. concerned, switch

E. endangered, click

 

Q. 170 The high cutoff marks this year have ________ college admission-seekers to either ________ for lesser known colleges or change their subject preferences.

A. cajoled, ask

B. pressured, sit

C. forced, settle

D. strained, compromise

E. entrusted, wait

 

Questions: 171 – 185

Directions (Q 171-185) :

Read the following passage carefully and answer the given questions. Certain words/phrases have been given in quotes to help you locate them while answering some of the questions.

India is rushing headlong towards economic success and modernization, counting on high-tech industries such as information technology and biotechnology to “propel” the nation to prosperity. India’s recent announcement that it would no longer produce unlicensed inexpensive generic pharmaceuticals bowed to the realities of the World Trade Organisation while at the same time challenging the domestic drug industry to compete with the multinational firms. Unfortunately, its weak higher education sector constitutes the “Achilles’ heel” of this strategy. Its systematic disinvestment in higher education in recent years has yielded neither world-class research nor very many highly trained scholars, scientists or managers to sustain high-tech development. India’s main competitors – especially China, but also Singapore, Taiwan and South Korea – are investing in large and “differentiated” higher education systems. They are providing access to a large number of students at the bottom of the academic system while at the same time building some research-based universities that are able to compete with the world’s best institutions. The recent London Times Higher Education Supplement ranking of the world’s top 200 universities include three in china, three in Hong Kong, three in South Korea, one in Taiwan and one in India. These countries are positioning themselves for leadership in the knowledge-based economies of the coming era. There was a time when countries could achieve economic success with cheap labour and low-tech manufacturing. Low wages still help, but contemporary large-scale development requires a sophisticated and at least partly knowledge based economy. India has chosen that path, but will find a major “stumbling block” in its university system. India has significant advantages in the 21st century knowledge race. It has a large higher education sector – the third largest in the world in terms of number of students, after china and the United States. It uses English as a primary language of higher education and research. It has a long academic tradition. Academic freedom is respected. There are a small number of high-quality institutions, departments and centres that can form the basis of quality sector in higher education. The fact that the States, rather than the Central Government, exercise major responsibility for higher education creates a rather “cumbersome” structure, but the system allows for a variety of policies and approaches. Yet the weakness far outweighs the strengths. India educates approximately 10 percent of its young people in higher education compared to more than half in the major industralised countries and 15 percent in china. Almost all of the world’s academic systems “resemble” a pyramid, with a small, high quality tier at the top and a “massive” sector at the bottom. India has a tiny top tier. None of its universities occupies a solid position at the top. A few of the best universities have some excellent departments and centres and there are a small number of outstanding undergraduate colleges. The University Grants Commission’s recent major support to five universities to build on their recognized strength is a step towards recognizing a differentiated academic system and “fostering” excellence. These universities, combines, enroll well under one percent of the student population.

 

Q. 171 Which of the following is TRUE in the context of the passage?

A. The top five universities in India educate more than 10 percent of the Indian student population

B. India’s higher education sector is the largest in the world

C. In the past, countries could progress economically through low manufacturing cost as well as low wages of labourers

D. India has recently invested heavy sums in the higher education sector leading to world-class research

E. All are true

 

Q. 172 What does the phrase “Achilles’ heel” mean as used in the passage?

A. Weakness

B. Quickness

C. Low quality

D. Nimbleness

E. Advantage

 

Q. 173 Which of the following is/are India’s strength/s in terms of higher education?

(A) Its system of higher education allows variations

(B) Medium of instruction for most higher learning is English

(C) It has the paraphernalia, albeit small in number, to build a high-quality higher

educational sector

A. Only (B)

B. Only (A) and (B)

C. Only (C)

D. Only (B) and (C)

E. All (A), (B) and (C)

 

Q. 174 What are the Asian countries, other than India, doing to head towards a knowledge based economy?

(A) Building competitive research-based universities

(B) Investing in diverse higher education systems

(C) Providing access to higher education to select few students

A. Only (A)

B. Only (A) and (B)

C. Only (B) and (C)

D. Only (B)

E. All (A), (B) and (C)

 

Q. 175 Which of the following is possibly the most appropriate title for the passage?

A. The Future of Indian Universities

B. Methods of overcoming the Educational Deficit in India

C. India and the Hunt for a Knowledge-Based Economy

D. Indian Economy Versus Chinese Economy

E. Indian Economy and its Features

 

Q. 176 What did India agree to do at the behest of the World Trade Organisation?

A. It would stop manufacturing all types of pharmaceuticals

B. It would ask its domestic pharmaceutical companies to compete with the international ones

C. It would buy licensed drugs only from the USA

D. It would not manufacture cheap common medicines without a license

E. None of these

 

Q. 177 Which of the following is/are India’s weakness/es when it comes to higher education?

(A) Indian universities do not have the requisite teaching faculty to cater to the needs of the higher education sector

(B) Only five Indian universities occupy the top position very strongly in the academic pyramid when it comes to higher education

(C) India has the least percentage of young population taking to higher education as compared to the rest of the comparable countries

A. Only (A) and (B)

B. Only (B)

C. Only (C)

D. Only (A) and (C)

E. All (A), (B) and (C)

 

Q. 178 Which of the following according to the passage is/are needed for the economic success of a country?

(A) Cheap labour

(B) Educated employees

(C) Research institutions to cater to the needs of development

A. Only (A) and (B)

B. Only (B)

C. Only (C)

D. Only (B) and (C)

E. All (A), (B) and (C)

 

Q. 179 Which word is most similar in meaning to the word “FOSTERING” as used in the passage?

A. Safeguarding

B. Neglecting

C. Sidelining

D. Nurturing

E. Ignoring

 

Q. 180 Which word is most similar in meaning to the word “PROPEL” as used in the passage?

A. Drive

B. Jettison

C. Burst

D. Acclimatize

E. Modify

 

Q. 181 Which word is most similar in meaning to the word “MASSIVE” as used in the passage?

A. Lump sum

B. Strong

C. Little

D. Gaping

E. Huge

 

Q. 182 Which word is most similar in meaning to the word “STUMBLING BLOCK” as used in the passage?

A. Argument

B. Frustration

C. Advantage

D. Hurdle

E. Fallout

 

Q. 183 Which word is opposite in meaning to the word “CUMBERSOME” as used in the passage?

A. Handy

B. Manageable

C. Breathtaking

D. Awkward

E. Difficult

 

Q. 184 Which word is opposite in meaning to the word “RESEMBLE” as used in the passage?

A. Against

B. Similar to

C. Mirror

D. Differ from

E. Unfavorable to

 

Q. 185 Which word is opposite in meaning to the word “DIFFERENTIATED” as used in the passage?

A. Similar

B. Varied

C. Harmonized

D. Synchronized

E. Discriminated

 

Questions: 186 – 190

Directions (Q 186-190) :

Rearrange the following six sentences (A), (B), (C), (D), (E) and (F) in the proper sequence to form a meaningful paragraph; then answer the questions that follow.

(A) Its prevalence reflects very badly on a society that is not able to stop this evil.

(B) Though elimination of child labour is an impossible task considering the current socio-economic scenario of these poor families, the Indian government is committed to the task of ensuring that no child remains illiterate, hungry and without medical care

(C) Therefore, unless the socio-economic status of the poor families is improved, India has to live with child labour.

(D) The members of these households have to send their children to work, even if the future of these innocent children is ruined, as that is the only choice open for them to survive in this world.

(E) Child labour is, no doubt, an evil that should be done away with at the earliest.

(F) But in a society where many households may have to suffer the pangs of hunger if the children are withdrawn from work, beggars can’t be choosers.

 

Q. 186 Which of the following should be the FIRST sentence after rearrangement?

A. A

B. E

C. C

D. F

E. D

 

Q. 187 Which of the following should be the THIRD sentence after rearrangement?

A. B

B. A

C. C

D. F

E. E

 

Q. 188 Which of the following should be the SECOND sentence after rearrangement?

A. A

B. F

C. D

D. B

E. C

 

Q. 189 Which of the following should be the FIFTH sentence after rearrangement?

A. E

B. A

C. F

D. C

E. D

 

Q. 190 Which of the following should be the SIXTH (LAST) sentence after rearrangement?

A. A

B. C

C. E

D. D

E. B

 

Questions: 191 – 200

Directions (Q 191-200) :

In the following passage, there are blanks, each of which has been numbered.

These numbers are given in the questions below and five words are given against it, one of which fits in the blank appropriately. Find out the appropriate word in each case. Around the world, forests are being (__1__) at a rate of about thirteen million hectares a year and deforestation accounts for an estimated 17-20% of all global emissions. In addition, forests and other terrestrial carbon sinks play a (__2__) role in preventing runaway climate change, soaking up a full 2.6 Gt of atmospheric carbon every year. The destruction of forests, therefore, not only emits carbon – a staggering 1.6 Gt a year, which severely (__3__) forest’s capacity to absorb emissions from other sources – but also drastically (__4__) the amount of forested land available to act as a carbon sink in the future. However, the effects of deforestation extend beyond carbon. Rainforests (__5__) a wide variety of ecosystems services, from regulating rainfall to purifying groundwater and keeping fertile soil from (__6__); deforestation in one area can seriously damage food production and (__7__) to clean water in an entire region. The value of global ecosystem services has been estimated at 33 trillion USD each year (almost half of global GDP), but these services have been taken for granted without a mechanism to make the market reflect their value. Rainforests are also a home and (__8__) of income for a huge number of people in Africa, Asia and South America. (__9__) this, economic pressures frequently drive both local communities and national governments in the developing world to (__10__) these forests in ways that are unsustainable, clear-cutting vast areas for fuel, timber, mining, or agricultural land.

 

Q. 191 The word suitable for (__1__) is

A. Ended

B. Destroyed

C. Extinct

D. Killed

E. Wasted

 

Q. 192 The word suitable for (__2__) is

A. Tough

B. Important

C. Vital

D. Biggest

E. Effective

 

Q. 193 The word suitable for (__3__) is

A. Affects

B. Diminishes

C. Increases

D. Alternates

E. Impairs

 

Q. 194 The word suitable for (__4__) is

A. Plagues

B. Develops

C. Reduces

D. Shortens

E. Influences

 

Q. 195 The word suitable for (__5__) is

A. Sell

B. Offer

C. Give

D. Provide

E. Earns

 

Q. 196 The word suitable for (__6__) is

A. Transforming

B. Decoding

C. Erupting

D. Draining

E. Eroding

 

Q. 197 The word suitable for (__7__) is

A. Handiness

B. Excess

C. Availability

D. Access

E. Supply

 

Q. 198 The word suitable for (__8__) is

A. Beginning

B. Source

C. Ways

D. Reference

E. Measure

 

Q. 199 The word suitable for (__9__) is

A. Despite

B. Also

C. Inspite

D. Apart

E. Beside

 

Q. 200 The word suitable for (__10__) is

A. Exploit

B. Encompass

C. Nurture

D. Work

E. Improve

 

 

Answer Sheet
Question 1 2 3 4 5 6 7 8 9 10
Answer B D A D A E D A B B
Question 11 12 13 14 15 16 17 18 19 20
Answer C E C B C C D E C D
Question 21 22 23 24 25 26 27 28 29 30
Answer D E A D C E B C E B
Question 31 32 33 34 35 36 37 38 39 40
Answer D C D B D A A A E A
Question 41 42 43 44 45 46 47 48 49 50
Answer D A A A B D A D D B
Question 51 52 53 54 55 56 57 58 59 60
Answer E B C A D D B A E D
Question 61 62 63 64 65 66 67 68 69 70
Answer A C E B D C A D E D
Question 71 72 73 74 75 76 77 78 79 80
Answer C E D E C B C B A E
Question 81 82 83 84 85 86 87 88 89 90
Answer C A D C E D A E C C
Question 91 92 93 94 95 96 97 98 99 100
Answer B D A E C C A B D E
Question 101 102 103 104 105 106 107 108 109 110
Answer B B A A E A A C D E
Question 111 112 113 114 115 116 117 118 119 120
Answer E E C A B D A B C D
Question 121 122 123 124 125 126 127 128 129 130
Answer D C C B D D B B C A
Question 131 132 133 134 135 136 137 138 139 140
Answer C E A D C E C C E A
Question 141 142 143 144 145 146 147 148 149 150
Answer C C B C A D D A D A
Question 151 152 153 154 155 156 157 158 159 160
Answer C B E B A B A C E C
Question 161 162 163 164 165 166 167 168 169 170
Answer E A E C B D B A D C
Question 171 172 173 174 175 176 177 178 179 180
Answer C A D B C D C E D A
Question 181 182 183 184 185 186 187 188 189 190
Answer E D B D A B D A D E
Question 191 192 193 194 195 196 197 198 199 200
Answer B C E C B E D B A A

 

IBPS SO Prelims 2017 Previous Paper

IBPS SO Prelims 2017

Direction (1-5): Study the information given below and answer the questions based on it. friends A, I, C, D, E, F, G and H are sitting in a straight line facing north and southdirection. Among them four are facing south, while four are facing north. They play different games Swimming, Cricket, Kabaddi, Hockey, Tennis, Volleyball, Basketball and Chess. The following information is given about them:

1) I who plays chess sits exactly between D and the person who plays Kabaddi.
2) The person who plays tennis faces north and is to the immediate right of the person who plays Swimming.
3) C and D are not the immediate neighbours of G. They do not sit at the extreme ends of the row.
4) The person who plays hockey sits at one of the ends and is second to the right of G who faces south. A does not like to play cricket.
5) The person who plays cricket is third to the left of the person who plays volleyball.
6) Also, C does not play Kabaddi. E faces south.
7) H is second to the right of the person who plays basketball.
8) A is to the immediate right of the person who faces north.
9) Exactly one person sits between D and C. C faces south.

Question 1. The person to the immediate left of C plays which of the following games?
A. Swimming

B. Cricket

C. Tennis

D. Basketball

E. Kabaddi

Question 2. Who sits exactly between I and the person who plays volleyball?
A. G

B. D

C. H

D. F

E. None of these

Question 3. Which of the following statements is true?
A. The person second to the left of D faces north

B. Exactly one person sits between I and C

C. The person to the immediate right of A plays hockey.

D. F faces south

E. None of these

Question 4. Who plays cricket?
A. D

B. E

C. G

D. F

E. A

Question 5. Four of the following five are alike and hence form a group. Which of the following five does not belong to the group?
A. FA

B. DH

C. CE

D. IG

E. AD

Direction (6-9): Read the following information carefully and answer the questions that follow:
P, Q, R, S, T, V and W are seven different boxes of different colours i.e. black, silver, red, pink, yellow, white and green but not necessarily in the same order. Box S is immediately above yellow colour box. More than three boxes are there between pink and silver colour box. There are two boxes between the box Q and box T. There is only one box between box T and box W. There are three boxes between box W and box P, which is of black colour. There are only two boxes between box P and box R, which is of white colour. The silver colour box is immediately above the box W. More than three boxes are there between yellow and pink colour box. T is of red colour box.

Question 6. How many boxes are there between green colour box and silver colour box?
A. One

B. Two

C. Three

D. Four

E. None

Question 7. Box Q is of which colour?
A. Silver

B. Green

C. Yellow

D. Black

E. Pink

Question 8. Which box is placed immediately above red colour box?
A. P

B. S

C. R

D. T

E. W

Question 9. How many boxes are there between box W and box Q?
A. None

B. One

C. Two

D. Three

E. Four

 Direction: Read the following information carefully and answer the questions that follow:

Seven persons Tina, Vini, Yasir, Rishi, Sanya, Pankaj and Urmila live on the separate floors of a 7- floor building. Ground floor is number one; second floor is number two and so on. Each of them goes to a city viz. Paris, Istanbul, Shanghai, Durban, London, Madrid and Dubai but not necessarily in the same order.

Only three people live above the floor on which Sanya lives. Only one person lives between Sanya and the one who goes to Paris. Vini lives just below the person who goes to Madrid. Only three people live between the one who goes to Paris and London. The person who goes to Madrid lives on an even numbered floor. Urmila lives just above Rishi. Urmila does not go to London.Only two persons live between Pankaj and the one who goes to Durban. Pankaj lives above the person who goes to Durban. Yasir does not go to Istanbul. Tina does not live just above or just below Sanya. The one who goes to Shanghai does not live just above or just below Pankaj.

Question 10. Who among the following lives on floor number 7?
A. Tina

B. Yasir

C. Pankaj

D. Urmila

E. Vini

Direction (11-15): Read the following information carefully and answer the questions given below it. Seven persons A, U, S, T, J, Z and M live in four different floors. The lowermost floor is floor number 1 and the topmost floor is floor
number 4. Two persons live on each floor while only one person lives on any of the floor. They go to movies in different months viz. January, February, March and April. They go to movies either on 13th of the month or on 27th of the month. Two persons go on each month while only one person go in any of the month. A lives neither on floor 1 nor on floor 4. M goes to movie in a month which has the least number of days but not on 27th. U does not go to movie on 13th of any month. Only one person lives on floor 3 and he goes to movies in February. J and Z go to movies in the same month but not in January. They live on same floor. A does not go to movie on 13th of any month. U does not live above M. Only one person goes to movie in February. S and T go to movies in different months. S does not live with U or T. T goes in March along with A. J goes after Z. J and Z can’t be on first floor. S does not go to movie on 27th of any month. T goes on 13th of the month.

Question 11. Who goes to movies in April?
A. Z and J

B. S and U

C. T and A

D. M and S

E. T and M

Question 12. Find the odd man out.
A. Z

B. J

C. M

D. A

E. T

Question 13. When does S go to the movie?
A. 27th January

B. 13th April

C. 13th March

D. 27th April

E. 13th January

Question 14. How many persons go to movie between J and U?
A. One

B. Three

C. More than three

D. Two

E. None

Question 15. Who goes fourth to watch the movie?
A. M

B. A

C. T

D. U

E. S

×

Hello!

Click one of our representatives below to chat on WhatsApp or send us an email to info@vidhyarthidarpan.com

×